109
GOALTIDE IAS ACADEMY GOALTIDEIAS.COM | [email protected] QUIZ COMPILATIONS – JUNE PART 1

QUIZ COMPILATIONS – JUNE PART 1

  • Upload
    others

  • View
    6

  • Download
    0

Embed Size (px)

Citation preview

Page 1: QUIZ COMPILATIONS – JUNE PART 1

GOALTIDE IAS ACADEMY GOALTIDEIAS.COM | [email protected]

QUIZ COMPILATIONS – JUNE PART 1

Page 2: QUIZ COMPILATIONS – JUNE PART 1

GOALTIDE IAS ACADEMY 1

QUIZ COMPILATIONS – JUNE PART 1

Q1. Consider the following statements regarding Deputy Speaker.

1. The House of the People shall choose deputy speaker within one month of assemble

of the house.

2. It is the duty of the Speaker to fix the date and initiate election of Deputy Speaker.

3. It is a parliamentary convention in India that the Deputy Speaker is elected from the

opposition parties.

Which of the above statements is/are correct?

a. 1 and 2 only

b. 3 only

c. 1 and 3 only

d. 2 and 3 only

Solution: d

Explanation:

Why this topic is important today:

It is the first time in the history of Indian republic that the office of Deputy speaker has

been vacant for almost two years (as of May, 2021). Ab dimag khol lijye, kuch important

aane waala h.

First statement is incorrect:

Article 93 of the Constitution of India provides that the House of the People shall, as soon

as may be, choose two members of the House to be respectively Speaker and Deputy

Speaker thereof and, so often as the office of Speaker or Deputy Speaker becomes vacant,

the House shall choose another member to be Speaker or Deputy Speaker.

Article 93 of the Constitution of India expressly does not put any rigid limitation in the

election of the Speaker and Deputy Speaker by the House of the People. However, in order

to balance the time limit and discretion of the members of the House of the People in

electing the Speaker and Deputy Speaker, they used the expression "as soon as may". The

expression "as soon as may be" is not defined under the Indian Constitution. First statement

is incorrect. ONE MONTH IS WRONG.

One more thing we will add here that: For president, framers of the Constitution has

explicitly mentioned time limit under Article 62 (2).

Page 3: QUIZ COMPILATIONS – JUNE PART 1

GOALTIDE IAS ACADEMY 2

QUIZ COMPILATIONS – JUNE PART 1

Second statement is correct:

The elaborate procedure for the election is provided under Rule 8 of the Rules of Procedure

and Conduct of Business in Lok Sabha. It provides that the election of a Deputy Speaker shall

be held on such date as the Speaker may fix, and the Secretary-General shall send to every

member notice of this date. It becomes clear here that the first duty to start the procedure

for election of Deputy Speaker is casted on the Speaker himself.

Third statement is correct:

It is a parliamentary convention in India that the Deputy Speaker is elected from the

opposition parties (there have been fifteen Deputy Speakers in India since independence,

almost all of them from opposition parties). This has led opposition parties to raise doubts

over the integrity of the ruling party and Speaker of the House of the People. Vacancy in the

office of the Deputy Speaker is an anathema to the principles of democracy.

Q2. World is celebrating “World Milk Day” on June 1, 2021. We will solve one question.

Consider the following statements.

1. India is the largest milk producing country in the world.

2. One of the objectives of Rashtriya Gokul Mission is to enhance milk production and

productivity.

3. Despite increasing milk production, per capita availability of milk in the country

currently is less than 1950-51.

Which of the above statements is/are correct?

a. 1 and 2 only

b. 2 and 3 only

c. 1 only

d. 1, 2 and 3

Solution: a

Explanation:

First statement is correct:

India ranks first among the world’s milk producing Nations since 1998 and has the largest

bovine population in the World. Milk production in India during the period 1950-51 to 2017-

Page 4: QUIZ COMPILATIONS – JUNE PART 1

GOALTIDE IAS ACADEMY 3

QUIZ COMPILATIONS – JUNE PART 1

18, has increased from 17 million tonnes to 176.4 million tonnes as compared to 165.4

million tonnes during 2016-17 recording a growth of 6.65 %.

Second statement is correct:

Third statement is incorrect:

The per capita availability of milk in the country which was 130 gram per day during 1950-

51 has increased to 374 gram per day in 2017-18 as against the world estimated average

consumption of 294 grams per day during 2017. This represents sustained growth in the

availability of milk and milk products for our growing population.

Link: http://dahd.nic.in/about-us/divisions/cattle-and-dairy-development

Q3. Consider the following statements regarding composition of National Human Rights

Commission.

1. The chairperson of NHRC shall be the retired CJI, not any SC judge.

2. One member of the commission having knowledge of human rights shall be woman.

3. Chairperson of National Commission for the Protection of Child Rights is the member

of the Commission.

Select the correct code.

a. 1 and 2 only

b. 2 and 3 only

c. 2 only

d. 3 only

Page 5: QUIZ COMPILATIONS – JUNE PART 1

GOALTIDE IAS ACADEMY 4

QUIZ COMPILATIONS – JUNE PART 1

Solution: b

Explanation:

The Protection of Human Rights (Amendment) Bill, 2019 brought several changes in the

composition.

The Protection of Human Rights (Amendment) Bill, 2019, inter alia, provides

a. that a person who has been a Judge of the Supreme Court is also made eligible to be

appointed as Chairperson of the Commission in addition to the person who has been

the Chief Justice of India;

b. to increase the Members of the Commission from two to three of which, one shall be a

woman;

c. to include Chairperson of the National Commission for Backward Classes, Chairperson

of the National Commission for Protection of Child Rights and the Chief Commissioner

for Persons with Disabilities as deemed Members of the Commission;

d. to reduce the term of the Chairperson and Members of the Commission and the State

Commissions from five to three years and shall be eligible for re-appointment;

e. to provide that a person who has been a Judge of a High Court is also made eligible to

be appointed as Chairperson of the State Commission in addition to the person who

has been the Chief Justice of the High Court; and,

f. to confer upon State Commissions, the functions relating to human rights being

discharged by the Union territories, other than the Union territory of Delhi, which will

be dealt with by the Commission.

Link: https://prsindia.org/billtrack/the-protection-of-human-rights-amendment-bill-2019

Q4. Consider the following statements regarding Federation of Indian Chambers of

Commerce & Industry.

1. It is a non-government, not-for-profit organization formed during Indira Gandhi

Government.

2. By engaging with the policy makers, government and civil society FICCI influences

the policies by way of articulating the views and suggestions of industry.

3. “Business Confidence Survey” is a publication of FICCI.

Which of the above statements is/are correct?

a. 1 and 2 only

b. 2 and 3 only

Page 6: QUIZ COMPILATIONS – JUNE PART 1

GOALTIDE IAS ACADEMY 5

QUIZ COMPILATIONS – JUNE PART 1

c. 2 only

d. 1, 2 and 3

Solution: b

Explanation:

Third statement is correct:

FICCI has published its quarterly “Business Confidence Survey”. According to it, Business

confidence of Indian companies has come down to its lowest in three quarters in May. In

last survey, confidence was at decadal high.

First statement is incorrect:

Established in 1927, FICCI is the largest and oldest apex business organization in India. Its

history is closely interwoven with India's struggle for independence, its industrialization, and

its emergence as one of the most rapidly growing global economies.

Second statement is correct:

A non-government, not-for-profit organization, FICCI is the voice of India's business and

industry. From influencing policy to encouraging debate, engaging with policy makers and

civil society, FICCI articulates the views and concerns of industry. It serves its members from

the Indian private and public corporate sectors and multinational companies, drawing its

strength from diverse regional chambers of commerce and industry across states, reaching

out to over 2,50,000 companies.

Q5. Tropic of Capricorn passes through:

a. Kalahari Desert

b. Rocky Mountain

c. Sahara Desert

d. Gobi Desert

Solution: a

Explanation:

You should know:

a. Where are these deserts?

b. Tropic of CAPRICORN

Answer is Kalahari Desert. Rocky is in US, Sahara in North Africa, Gobi in China. Tropic of

Capricorn passes through southern part of Africa, you can easily eliminate all.

Page 7: QUIZ COMPILATIONS – JUNE PART 1

GOALTIDE IAS ACADEMY 6

QUIZ COMPILATIONS – JUNE PART 1

Q6. In the present India context, right to work is a:

a. Constitutional right

b. Legal right

c. Natural Right

d. None of the above

Solution: b

Explanation:

Page 8: QUIZ COMPILATIONS – JUNE PART 1

GOALTIDE IAS ACADEMY 7

QUIZ COMPILATIONS – JUNE PART 1

Directive Principles of State Policy (article 41) directs the state to secure Right to work, to

education and to public assistance in certain cases The State shall, within the limits of its

economic capacity and development, make effective provision for securing the right to

work, to education and to public assistance in cases of unemployment, old age, sickness and

disablement, and in other cases of undeserved want.

And we know DPSP is not enforceable. Hence, it is not a constitutional right.

It is a legal right. Example, MNREGA. In MNREGA (statutory status), it is stated that 100 days

of work is guaranteed. Right to work is a legal right.

Q7. With reference to the religious practices in India, the “Taran Panth” sect belongs to:

a. Buddhism

b. Jainism

c. Vaishnavism

d. Shaivism

Solution: b

Explanation:

In 2018, UPSC asked similar question related to Sthanakvasi sect.

The Taran Panth, is a sect of Digambar Jainism founded by Taran Swami in Bundelkhand in

central India.

Q8. Consider the following Statements.

1. Elections disputes pertaining to Panchayats shall be decided in accordance with law

of Parliament.

2. As per the Constitution, Audit and Accounts of Panchayats is done by State Finance

Commission.

Which of the above statement is correct?

a. 1 only

b. 2 only

c. Both 1 and 2

Page 9: QUIZ COMPILATIONS – JUNE PART 1

GOALTIDE IAS ACADEMY 8

QUIZ COMPILATIONS – JUNE PART 1

d. Neither 1 nor 2

Solution: d

Explanation:

Elections disputes pertaining to Panchayats shall be decided in accordance with law of

State Legislature. Therefore, statement 1 is wrong.

Audit and Account of State Legislature is done in accordance with law of state legislature.

Therefore, statement 2 is wrong.

Q9. The term “One Country Two Systems” is sometimes mentioned in the news in the

context of affairs of:

a. Israel

b. China

c. Pakistan

d. Yemen

Solution: b

Explanation:

Hong Kong was ceded to the UK after a war in 1842.

Britain and China began talks on the future of Hong Kong - with the communist government

in China arguing that all of Hong Kong should be returned to Chinese rule.

The two sides reached a deal in 1984 that would see Hong Kong return to China in 1997,

under the principle of "one country, two systems".

A similar question was asked in UPSC Prelims 2018:

Page 10: QUIZ COMPILATIONS – JUNE PART 1

GOALTIDE IAS ACADEMY 9

QUIZ COMPILATIONS – JUNE PART 1

Q10. "Hindola Raga"-A famous paintings belongs to:

a. Ahmednagar

b. Bijapur

c. Golconda

d. Tanjore

Solution: a

Explanation:

It Belongs to Ahmednagar.

Link: http://ccrtindia.gov.in/miniaturepainting.php

Q11. Consider the following statements.

1. Once the nominated person resigns from the Rajya Sabha, as per the Constitution,

he can be renominated to the house for the remainder period.

2. Elected and nominated members of Legislative Assembly elect the representatives

for Rajya Sabha.

3. The allocation of seats in the Council of States is in accordance with fourth schedule.

Which of the above statements is/are correct?

a. 1 and 3 only

b. 3 only

c. 2 and 3 only

d. 1, 2 and 3

Solution: b

Explanation:

Former journalist Swapan Dasgupta, who had resigned from Rajya Sabha and unsuccessfully

contested the recent West Bengal assembly elections on a Bharatiya Janata Party ticket, has

been renominated to the upper house of parliament by the government.

Page 11: QUIZ COMPILATIONS – JUNE PART 1

GOALTIDE IAS ACADEMY 10

QUIZ COMPILATIONS – JUNE PART 1

Under Article 80(1)(a) there is a nomination. But there is no renomination. The president

can only nominate a person to the Raya Sabha.

Also, Section 154(3) of the Representation of People Act says that a member chosen to fill

a casual vacancy shall serve the remainder of the term of his predecessor. Mark the word

‘chosen’, which means chosen through election. It does not apply to a nominated member.

It would therefore mean that filling a casual vacancy does not apply to nomination. So, the

notification issued by the president re-nominating Dasgupta seems to be defective (Court

will decide if PIL is filed, but as per the provisions, it is not mentioned anywhere that he

can be renominated.)

Second statement is incorrect:

Only elected, NOT NOMINATED.

Third statement is correct: See the first image, 80 (2).

Q12. Consider the following statements regarding International Renewable Energy Agency

(IRENA).

1. IRENA was the first international organization to focus exclusively on renewable

energies, addressing the needs of both the industrialized and developing worlds.

2. The agency is headquartered in Abu Dhabi, UAE.

3. India is a member of IRENA.

Which of the above statements is/are correct?

a. 1 and 2 only

b. 2 and 3 only

c. 1 and 3 only

d. 1, 2 and 3

Solution: d

Explanation:

Page 12: QUIZ COMPILATIONS – JUNE PART 1

GOALTIDE IAS ACADEMY 11

QUIZ COMPILATIONS – JUNE PART 1

First statement is correct:

The International Renewable Energy Agency (IRENA) is an intergovernmental organisation

mandated to facilitate cooperation, advance knowledge, and promote the adoption and

sustainable use of renewable energy.

Second statement is correct:

Third statement is correct:

India is a member of IRENA.

Q13. Consider the following statements regarding Comptroller and Auditor-General of India.

1. CAG shall only be removed from office in like manner and on the like grounds as a

Judge of the Supreme Court.

2. The duties and Powers are CAG are mentioned in the Constitution.

Which of the above statements is/are correct?

a. 1 only

b. 2 only

c. Both 1 and 2

d. Neither 1 nor 2

Solution: a

Explanation:

Article 148 states there shall be a Comptroller and Auditor-General of India who shall be

appointed by the President by warrant under his hand and seal and shall only be removed

from office in like manner and on the like grounds as a Judge of the Supreme Court.

Page 13: QUIZ COMPILATIONS – JUNE PART 1

GOALTIDE IAS ACADEMY 12

QUIZ COMPILATIONS – JUNE PART 1

The Constitution has given authority to Parliament to frame power and duties of CAG.

So, parliament enacted the comptroller and auditor general's (Duties, Powers and

Conditions of Service) Amendment ACT, 1971.

Q14. Payment and Settlement systems in India is regulated by:

a. Department of Economic Affairs, Ministry of Finance

b. Reserve Bank of India

c. Department of Financial Services, Ministry of Finance

d. A committee of 6 members consists of RBI and Government in equal proportion.

Solution: b

Explanation:

The Payment and settlement system refer to the system which facilitates the transfer of

money from a payer to the beneficiary. It includes both paper-based payments (Cheques,

Drafts) as well as electronic payments such as Real Time Gross Settlement (RTGS), National

Electronic Funds Transfer (NEFT), Immediate payment Service (IMPS), UPI etc.

RBI regulates Payment and Settlement systems in India.

Page 14: QUIZ COMPILATIONS – JUNE PART 1

GOALTIDE IAS ACADEMY 13

QUIZ COMPILATIONS – JUNE PART 1

Q15. In which of the following way can the government finance its investment expenditure

such that there is minimal inflation?

1. Foreign aid

2. Deficit Financing

3. Taxation

4. Public Borrowing

Select the correct code.

a. 3 and 4 only

b. 1 and 2 only

c. 2 and 4 only

d. 1 and 3 only

Solution: a

Explanation:

Simple concept is: For minimizing inflation, we should minimize pumping of money into the

system.

In the above given 4 options, both foreign and deficit financing will result in injecting more

money into the system, resulting in inflation. On the other hand, adopting public borrowing

and taxation for the government’s investment expenditure, new money will not be

pumped into the system and hence the inflationary effect in this case will be minimum.

Q16. Consider the following Pairs.

Conventions/Protocols Related to

1. Stockholm Persistent Organic Pollutants

2. Cartagena Advance Informed Agreement

procedures for transfer of

Living Modified Organisms

3. Basel Prior Informed Consent

Procedure for Certain

Hazardous Chemicals in

International Trade

Page 15: QUIZ COMPILATIONS – JUNE PART 1

GOALTIDE IAS ACADEMY 14

QUIZ COMPILATIONS – JUNE PART 1

Which of the above pairs is/are correct?

a. 1 and 2 only

b. 2 and 3 only

c. 1 and 3 only

d. 1. 2 and 3

Solution: a

Explanations:

Conventions/Protocols Related to

1. Stockholm Persistent Organic Pollutants

2. Cartagena Advance Informed Agreement

procedures for transfer of

Living Modified Organisms

3. Rotterdam Prior Informed Consent

Procedure for Certain

Hazardous Chemicals in

International Trade

4. Basel Control of Transboundary

Movements of Hazardous

Wastes and their Disposal

Q17. Which of the following defines ‘Project Artemis’ of NASA?

a. A moon exploration Mission

b. It observes the behavior of cosmos using the highest-energy form of light.

c. To study Venus

d. To study Asteroids

Solution: a

Explanation:

Page 16: QUIZ COMPILATIONS – JUNE PART 1

GOALTIDE IAS ACADEMY 15

QUIZ COMPILATIONS – JUNE PART 1

Q18. Consider the following countries.

1. Malawi

2. Zimbabwe

3. Mozambique

4. South Sudan

5. Niger

Which of the above countries are land-locked?

a. 1, 4 and 5 only

b. 2, 4 and 5 only

c. 1, 2, 4 and 5 only

d. 1, 2, 3 and 4 only

Solution: c

Explanation:

Page 17: QUIZ COMPILATIONS – JUNE PART 1

GOALTIDE IAS ACADEMY 16

QUIZ COMPILATIONS – JUNE PART 1

Q19. Consider the following statements regarding Preamble of India.

1. In the Berubari Union case (1960), it was decided that Preamble is a part of the

Constitution.

2. The Preamble of India reveals the date of adoption of the Constitution.

3. So far, it has been amended once by 44th Constitutional Amendment Act, 1978.

Which of the above statements is/are correct?

a. 1 and 2 only

b. 2 and 3 only

c. 2 only

d. 1 and 3 only

Solution: c

Explanation:

Page 18: QUIZ COMPILATIONS – JUNE PART 1

GOALTIDE IAS ACADEMY 17

QUIZ COMPILATIONS – JUNE PART 1

In the Berubari Union case (1960), the Supreme Court said that the Preamble shows the

general purposes behind the several provisions in the Constitution and is thus a key to the

minds of the makers of the Constitution.

Further, where the terms used in any article are ambiguous or capable of more than one

meaning, some assistance at interpretation may be taken from the objectives enshrined in

the Preamble. Despite this recognition of the significance of the Preamble, the Supreme

Court specifically opined that Preamble is not a part of the Constitution. Supreme Court

held that the Preamble is an integral part of the Constitution.

But, In the Kesavananda Bharati case (1973), the Supreme court rejected the earlier

opinion and held that Preamble is a part of the Constitution.

So, when Supreme court of India held that preamble is an integral part of the Constitution of

India it automatically classified as amendable. It has been amended only once so far, in

1976, by 42nd Constitutional Amendment Act, which has added three new words- Socialist,

Secular and Integrity to the preamble.

It stipulates 26th November 1949, as the date for the adoption of the Constitution. See

below.

Q20. Consider the following statements regarding Bar Council of India.

1. It is a statutory Body.

2. Attorney General is the ex-officio member of Council.

Which of the above statements is/are correct?

a. 1 only

b. 2 only

c. Both 1 and 2

d. Neither 1 nor 2

Solution: c

Page 19: QUIZ COMPILATIONS – JUNE PART 1

GOALTIDE IAS ACADEMY 18

QUIZ COMPILATIONS – JUNE PART 1

Explanation:

The Bar Council of India was established by Parliament under the Advocates Act, 1961.

Therefore, it is a statutory body.

Attorney General is ex-officio member.

Q21. Consider the following statements.

1. The Reserve Bank of India transfers its surplus profits to the Government of India in terms of

the provisions of the Reserve Bank of India Act, 1934.

2. The amount of surplus money to be transferred to Government from RBI is decided by

Monetary Policy Committee.

3. Except few fiscal years, we have seen a decreasing trend in surplus transfer in last decade.

Which of the above statements is/are correct?

a. 1 and 2 only

b. 3 only

c. 1 and 3 only

d. 1 only

Solution: d

Explanation:

First statement is correct:

The Reserve Bank of India (RBI) transfers its surplus profits to the Government of India in terms of

the provisions of Section 47 of the Reserve Bank of India Act, 1934 which is as follows:

“After making provision for bad and doubtful debts, depreciation in assets, contributions to staff and

superannuation funds and for all matters for which provision is to be made by or under this Act or

which are usually provided for by bankers, the balance of the profits shall be paid to the Central

Government.”

If we use dividends instead of transfer/surplus, it can go wrong. So, be careful.

Page 20: QUIZ COMPILATIONS – JUNE PART 1

GOALTIDE IAS ACADEMY 19

QUIZ COMPILATIONS – JUNE PART 1

Why are these called transfers to the government, rather than dividends?

That is because the RBI is not a commercial organization like banks and other companies owned or

controlled by the government to pay a dividend to the owner out of the profit generated. Though it

was promoted as a private shareholders’ bank in 1935 with a paid-up capital of Rs 5 crore, the

government nationalised it in January 1949, making the sovereign the “owner”. What the RBI does is

transfer the surplus — excess of income over expenditure —to the government.

Second statement is incorrect:

To quickly recapitulate the context, the Bimal Jalan Committee had been set up primarily to go into

the question of what portion of its annual profits/surplus the RBI could transfer to the GOI every

year and how much it needed to plough back into its own balance sheet. The ploughed-back profits

are the bank’s ‘reserves’ which every healthy balance sheet builds up so as to take care of

unexpected and unforeseen exigencies. The CRB band of 6.5 -5.5% is a function of those profit

payout/retention recommendations. The central bank has chosen to maintain the lowest required

buffer.

Third statement is incorrect:

Page 21: QUIZ COMPILATIONS – JUNE PART 1

GOALTIDE IAS ACADEMY 20

QUIZ COMPILATIONS – JUNE PART 1

Q22. World Employment and Social Outlook is a publication of:

a. World Bank

b. International Labor Organization

c. UNCTAD

d. WTO

Solution: b

Explanation:

Global unemployment is expected to be at 205 million in 2022, surpassing the 2019 level of 187

million, according to a new report.

The jobs shortfall induced by the novel coronavirus disease (COVID-19) pandemic was 75 million in

2021 and is expected to be 23 million in 2022, warned the World Employment and Social Outlook:

Trends 2021 published by International Labour Organization (ILO).

Link: https://www.downtoearth.org.in/news/economy/global-unemployment-to-surge-to-205-

million-in-2022-ilo-77254

Q23. Which of the following factors are leading to Freshwater Salinization Syndrome (FSS)?

1. Human-accelerated weathering of infrastructure, rocks and soils

2. Sea-level rise and saltwater intrusion

3. Evaporative concentration of salt ions from hydrologic modifications and climate

4. Disturbance of vegetation and local groundwater hydrology

Select the correct code.

a. 1, 2 and 3 only

b. 3 and 4 only

c. 1 and 2 only

d. 1, 2, 3 and 4

Solution: d

Explanation:

News is of April 2021, but use koi fark ni padta. We find it important and we deliver. Don’t bother

too much about date. All important topics will be covered if you do Daily Quiz and Tests Series

religiously.

Introducing salt into the environment — for de-icing roads, fertilizing farmlands and other purposes

— releases toxic chemicals that pose a threat to freshwater supply system. This is known as

freshwater salinization syndrome (FSS) or the effects of introduced salts can poison drinking water

and increase chloride concentrations over time, according to a recent study published in journal

Biogeochemistry. It was carried out by researchers of University of Maryland.

According to the research, FSS is caused by:

• Road salts

• Human-accelerated weathering of infrastructure, rocks and soils

Page 22: QUIZ COMPILATIONS – JUNE PART 1

GOALTIDE IAS ACADEMY 21

QUIZ COMPILATIONS – JUNE PART 1

• Sea-level rise and saltwater intrusion

• Evaporative concentration of salt ions from hydrologic modifications and climate

• Disturbance of vegetation and local groundwater hydrology

Up to 220 million people globally are at risk of exposure to elevated levels of arsenic in groundwater, which can also be mobilized by saltwater intrusion primarily in Asia. The study noted the risks of exposure to co-occurring, multiple heavy metals in drinking water in developing countries. More than 57 per cent of India’s groundwater was contaminated with nitrate, fluoride and arsenic, according to an analysis of the government data in the State of India’s Environment in Figures, 2020. Groundwater in at least 249 districts in 18 states and Union territories was found to be saline, according to the SOE in figures. Groundwater was unfit for consumption since it affects the digestive system, raises blood pressure and hypertension.

Link: https://www.downtoearth.org.in/news/water/fresh-water-is-turning-saltier-flags-study-76529

Q24. Which of the following bodies are constituted under Environment Protection Act, 1986?

1. Central Ground Water Authority

2. Central Pollution Control Board

3. Central Water Commission

Select the correct code.

a. 1 and 2 only

b. 3 only

c. 1 and 3 only

d. 1 only

Solution: d

Explanation:

Central Water Commission

Central Water Commission is a premier Technical Organization of India in the field of Water

Resources and is presently functioning as an attached office of the Ministry of Jal Shakti.

The Commission is entrusted with the general responsibilities of initiating, coordinating and

furthering in consultation of the State Governments concerned, schemes for control, conservation

and utilization of water resources throughout the country, for purpose of Flood Control, Irrigation,

Navigation, Drinking Water Supply and Water Power Development.

Central Water Commission CWC is headed by a chairman, with the status of Ex-Officio Secretary to

the Government of India.

Central Ground Water Authority

Central Ground Water Authority has been constituted under Section 3 (3) of the Environment

(Protection) Act, 1986 to regulate and control development and management of ground water

resources in the country.

Page 23: QUIZ COMPILATIONS – JUNE PART 1

GOALTIDE IAS ACADEMY 22

QUIZ COMPILATIONS – JUNE PART 1

Central Pollution Control Board

The Central Pollution Control Board (CPCB), statutory organization, was constituted in September

1974 under the Water (Prevention and Control of Pollution) Act, 1974. Further, CPCB was entrusted

with the powers and functions under the Air (Prevention and Control of Pollution) Act, 1981.

Q25. Ladakh shares boundary with:

1. Pakistan

2. Afghanistan

3. China

4. Turkmenistan

Select the correct code.

a. 1 and 2 only

b. 1 and 3 only

c. 1, 2 and 3 only

d. 2, 3 and 4 only

Solution: c

Explanation:

Ladakh shares boundary with Pakistan, China and Afghanistan.

Page 24: QUIZ COMPILATIONS – JUNE PART 1

GOALTIDE IAS ACADEMY 23

QUIZ COMPILATIONS – JUNE PART 1

Q26. Consider the following statements.

1. The nomination paper of a candidate for the Presidential election has to be subscribed by at

least fifty electors as proposers.

2. No President in India has been elected twice.

3. The President shall vacate his office immediately after expiry of his term.

Which of the above statements is/are correct?

a. 1 and 3 only

b. 2 only

c. 1 and 2 only

d. 1 only

Solution: d

Explanation:

Ok, very important thing here.

Under the Constitution of India, there shall always be a President of India (See Article 52 of the

Constitution). He holds the highest elective office in the country and is elected in accordance with

the provisions of the Constitution and the Presidential and vice-Presidential Elections Act, 1952.

This Act you should know. UPSC has already asked Judges Inquiry Act, 1968. Be careful.

Page 25: QUIZ COMPILATIONS – JUNE PART 1

GOALTIDE IAS ACADEMY 24

QUIZ COMPILATIONS – JUNE PART 1

As per the Presidential and vice-Presidential Election Act, 1952, the nomination paper of a

candidate for the Presidential election has to be subscribed by at least fifty electors as proposers

and at least fifty electors as seconders.

But in case of Vice-President, 20 proposers and seconders. Remember this.

The President shall hold office for a term of 5 years from the date on which he enters upon his

office. He shall, however, continue to hold office notwithstanding the expiry of his term, until his

successor enters upon his office. [Article 56 of the Constitution of India].

Dr Rajendra Prasad won the first two elections, in 1952 and 1957.

Q27. Who among the following has the power to appoint acting judges of High Court?

a. Chief Justice of India

b. Chief Justice of the concerned High Court

c. Governor

d. President of India

Solution: d

Explanation:

Refer to article 224. President appoints additional and acting Judges.

Page 26: QUIZ COMPILATIONS – JUNE PART 1

GOALTIDE IAS ACADEMY 25

QUIZ COMPILATIONS – JUNE PART 1

Q28. Samantabhadra, Ksitigarbha and Akasagarbha are:

a. Books compilations of Buddhism Philosophy

b. They are the tax imposed by Gupta empire on various commodities

c. Jain texts which talk about how to achieve salvation

d. Bodhisattvas

Solution: d

Explanation:

Samantabhadra, Ksitigarbha and Akasagarbha are Bodhisattvas.

Ksitigarbha is the savior of the oppressed, the dying

Samantabhadra represents kindness and happiness

Akasagarbha denotes repository of void. Void here doesn’t indicate nothingness, but the mysterious

potentiality that gives rise to all phenomena.

Q29. Consider the following.

1. Vote on Account

2. Definition of Money Bills

3. Language to be used in Parliament.

Which of the above provisions are mentioned in Indian Constitution (not by Convention)?

a. 1 and 2 only

b. 2 and 3 only

c. 1 and 3 only

d. 1, 2 and 3

Solution: d

Explanation:

All provisions are mentioned in the Constitution.

Page 27: QUIZ COMPILATIONS – JUNE PART 1

GOALTIDE IAS ACADEMY 26

QUIZ COMPILATIONS – JUNE PART 1

Q30. With reference to the Constitution, Consider the following statements regarding Co-operative

societies.

1. The superintendence and control of the preparation of electoral rolls for elections to a co-

operative society shall vest in State Election Commission.

2. The Constitution provides for the maximum number of directors of a co-operative society.

Which of the following options is/are correct?

Page 28: QUIZ COMPILATIONS – JUNE PART 1

GOALTIDE IAS ACADEMY 27

QUIZ COMPILATIONS – JUNE PART 1

a. 1 only

b. 2 only

c. Both 1 and 2

d. Neither 1 nor 2

Solution: b

Explanation:

Keep in mind one basic rule, if you really want to understand polity in true sense, go through

provisions from Constitution books. You will be able to learn few more things.

Now get back to statements.

Article 243ZK (2)

The superintendence, direction and control of the preparation of electoral rolls for, and the

conduct of, all elections to a co-operative society shall vest in such an authority or body, as may be

provided by the Legislature of a State, by law:

Therefore, now you can clearly strike down option 1.

Now for second statement, refer to Article 243ZJ below:

The board shall consist of such number of directors as may be provided by the Legislature of a State,

by law:

Therefore, statement 2 is correct

Q31. Consider the following statements.

1. Voluntary National Review (VNR) by nations aims to accelerate the implementation of the

2030 Agenda.

2. India has produced VNR only once in 2017.

Page 29: QUIZ COMPILATIONS – JUNE PART 1

GOALTIDE IAS ACADEMY 28

QUIZ COMPILATIONS – JUNE PART 1

3. VNR action plan in India is prepared by NITI Aayog.

Which of the above statements is/are correct?

a. 1 and 2 only

b. 3 only

c. 1 and 3 only

d. 1 only

Solution: c

Explanation:

First statement is correct:

The voluntary national reviews (VNRs) aim to facilitate the sharing of experiences, including

successes, challenges and lessons learned, with a view to accelerating the implementation of the

2030 Agenda. The VNRs also seek to strengthen policies and institutions of governments and to

mobilize multi-stakeholder support and partnerships for the implementation of the Sustainable

Development Goals.

Second statement is incorrect. Third statement is correct.

Page 30: QUIZ COMPILATIONS – JUNE PART 1

GOALTIDE IAS ACADEMY 29

QUIZ COMPILATIONS – JUNE PART 1

NITI Aayog presented India’s second Voluntary National Review (VNR) at the United Nations High-

level Political Forum (HLPF) on Sustainable Development, 2020. The HLPF is the foremost

international platform for follow-up and review of progress on the 17 Sustainable Development

Goals (SDGs). The India VNR 2020 report titled Decade of Action: Taking SDGs from Global to Local

was released NITI Aayog.

Q32. National Indicator Framework (NIF) for measuring the progress of the SDGs and associated

targets at the national level is developed by:

a. NITI Aayog

b. Cabinet Secretariat

c. Ministry of Finance

d. Ministry of Statistics and Programme Implementation

Solution: d

Explanation:

MoSPI is responsible for the development of National Indicator Framework (NIF) for measuring the

progress of the SDGs and associated targets at the national level. NIF will give appropriate direction

to the policy makers and the implementers of various schemes and programs.

Q33. Consider the following Pairs.

Communities/groups Belongs to

1. Hazaras Afghanistan

2. Houthis Yemen

Which of the above pairs is/are correct?

a. 1 only

b. 2 only

c. Both 1 and 2

d. Neither 1 nor 2

Solution: d

Explanation:

Communities/groups Belongs to

Hazaras Afghanistan

Houthis Yemen

Q34. Consider the following statements:

1. As per law, the Compensatory Afforestation Fund Management and Planning Authority exists

at both National and State levels.

2. As per the law, National Compensatory Afforestation Fund will be created at both National and

State levels.

Page 31: QUIZ COMPILATIONS – JUNE PART 1

GOALTIDE IAS ACADEMY 30

QUIZ COMPILATIONS – JUNE PART 1

3. Whenever forest land is diverted for non-forest purposes, it is mandatory under the Forest

(Conservation) Act, 1980 that an equivalent area of non-forest land has to be taken up for

compensatory afforestation.

Which of the above statements is/are correct?

a. 1 and 2 only

b. 2 and 3 only

c. 1 only

d. 1, 2 and 3

Solution: d

Explanation:

First statement is correct:

Second statement is correct:

Third statement is correct:

Whenever forest land is diverted for non-forest purposes, it is mandatory under the Forest

(Conservation) Act, 1980 that an equivalent area of non-forest land has to be taken up for

compensatory afforestation.

Q35. Consider the following Central American Countries.

1. Costa Rica

2. Nicaragua

3. Panama

Page 32: QUIZ COMPILATIONS – JUNE PART 1

GOALTIDE IAS ACADEMY 31

QUIZ COMPILATIONS – JUNE PART 1

4. Guatemala

Arrange the above countries from West to East.

a. 2-1-3-4

b. 2-1-4-3

c. 4-2-1-3

d. 4-1-2-3

Solution: c

Explanation:

The correct order of countries from West to East:

Guatemala- Nicaragua- Costa Rica- Panama

Q36. Which of the following countries form border with both Atlantic Ocean and

Mediterranean Sea?

1. Portugal

2. France

3. Germany

4. Spain

Select the correct code.

a. 1, 2 and 4 only

b. 2 and 4 only

c. 1, 2 and 3 only

d. 3 and 4 only

Solution: b

Page 33: QUIZ COMPILATIONS – JUNE PART 1

GOALTIDE IAS ACADEMY 32

QUIZ COMPILATIONS – JUNE PART 1

Explanation:

Q37. Consider the following statements regarding Financial Emergency mentioned in Indian

Constitution.

1. The Financial Emergency drew inspiration from the National Recovery Act of the

United States, passed in the year 1933 to combat the aftereffects of the great

depression

2. During Financial Emergency, President may issue directions for the reduction of

salaries and allowances of Judges of the Supreme Court and the high court.

3. Financial Emergency (FM) has been imposed in our country once when the worst

financial crises hit our country in 1991.

Select the correct code.

a. 1 only

b. 1 and 2 only

c. 2 and 3 only

d. 1, 2 and 3

Solution: b

Explanation:

Very easy questions, if you know last two statements.

Article 360 titled 'Provisions as to financial emergency' enables:

a. The President to proclaim financial emergency, if he is satisfied that the financial

stability or credit of India or of any part of the territory thereof is threatened

Page 34: QUIZ COMPILATIONS – JUNE PART 1

GOALTIDE IAS ACADEMY 33

QUIZ COMPILATIONS – JUNE PART 1

b. The executive authority of the Union to direct any State to observe such canons of

financial propriety as may be specified in the directions

c. The executive authority of the Union to give "such other directions" to the States "as

the President may deem necessary and adequate for the purpose"; and

d. The President to issue directions for the reduction of salaries and allowances of all or

any class of persons serving in connection with the affairs of the Union including the

Judges of the Supreme Court and the High Courts.

Financial Emergency (FM) has never been imposed in our country even when the worst

financial crises hit our country in 1991. Third statement is wrong. So, now you are left with

options 1 and 2. 1 is in both options, so you should know second statement.

One of the debates of BR Ambedkar:

Dr. Ambedkar explained that draft Article 360 drew inspiration from the National Recovery

Act of the United States, passed in the year 1933 to combat the aftereffects of the great

depression. The National Recovery Act could never be effectively used, since it was struck

down by the United States Supreme Court, soon after its enactment. To prevent such a

situation from befalling upon any prospective Indian legislation, the framers of the

Constitution deemed it fit to introduce financial emergency provisions in the Constitution

itself.

Q38. With respect to the members of Legislative Council, consider the following

statements.

1. If a person elected as a member of Legislative Council declared as of unsound mind,

the final decision to disqualify him remains with Governor.

2. Every member of the Legislative Council of a State shall, before taking his seat, make

and subscribe before the Governor or person appointed by him.

Which of the above statements is/are correct?

a. 1 only

b. 2 only

c. Both 1 and 2

d. Neither 1 nor 2

Solution: c

Explanation:

Both statements are correct. Read three articles given below:

Second statement is correct:

Page 35: QUIZ COMPILATIONS – JUNE PART 1

GOALTIDE IAS ACADEMY 34

QUIZ COMPILATIONS – JUNE PART 1

First statement is correct.,

Q39. Which among the following are the work areas of IUCN (International Union for

Conservation of Nature and Natural Resources)?

1. Biodiversity Conservation

2. Poverty Alleviation

3. Gender Equality

Choose the correct answer using codes given below.

a. 1 only

b. 1 and 2 only

c. 1 and 3 only

d. 1,2 and 3

Solution: d

Explanation:

You have always read IUCN maintains Red list, so Biodiversity Conservation is good but

what about other?? All are included. Don’t get confused. If you don’t read topics from

Page 36: QUIZ COMPILATIONS – JUNE PART 1

GOALTIDE IAS ACADEMY 35

QUIZ COMPILATIONS – JUNE PART 1

original websites problem will come. We are trying to reduce your problem in all possible

ways. Just be consistent. Always try to read topics from original website.

Since its establishment in 1948, IUCN has become the global authority on the status of the

natural world and the measures needed to safeguard it. The knowledge and the tools IUCN

provide are critical for ensuring that human progress, economic development and nature

conservation take place together. IUCN is a membership Union composed of both

government and civil society organisations.

In the early 2000s, IUCN developed its business engagement strategy. Prioritising sectors

with a significant impact on nature and livelihoods, such as mining and oil and gas, its aim is

to ensure that any use of natural resources is equitable and ecologically sustainable.

Later in the 2000s, IUCN pioneered ‘nature-based solutions’ – actions to conserve nature

which also address global challenges, such as food and water security, climate change and

poverty reduction.

Q40. Consider the following:

1. Photosynthesis

2. Respiration

3. Decay of organic matter

4. Volcanic action

Which of the above add carbon dioxide to the carbon cycle on Earth?

a. 1 and 4 only

b. 2 and 3 only

c. 2, 3 and 4 only

d. 1, 2, 3 and 4

Solution: c

Explanation:

Photosynthesis takes out CO2 from carbon cycle. Rest all ads CO2. It’s like you can

understand easily, no need to explain all factors.

Every volcanic eruption that occurs on planet Earth is full of pollutants. Not just ash and

dust, mind you, but also carbon dioxide: one of the strongest greenhouse gases on our

planet.

Q41. Consider the following statements.

1. Currently, India’s installed renewable energy capacity is around 20 percent of the total

target announced in INDCs (Paris climate summit).

2. More than 50 percent of total installed renewable energy capacity is in western region of

India.

3. Renewable Energy is second largest source of installed capacity after thermal.

Page 37: QUIZ COMPILATIONS – JUNE PART 1

GOALTIDE IAS ACADEMY 36

QUIZ COMPILATIONS – JUNE PART 1

Which of the above statements is/are correct?

a. 1 and 3 only

b. 3 only

c. 2 and 3 only

d. 3 only

Solution: d

Explanation:

We will go to latest data and find answer to all statements. If not ask directly, these questions will

definitely help you in eliminating options and reaching final answer. So, practice sincerely.

First statement is incorrect:

Just before the Paris climate summit in 2015, the Government of India had said it would install 175

GW of renewable power by 2022, including 100 GW of solar power. The rest was targeted from wind

(60 GW) of bio-power (10 GW) and small hydropower (5 GW). At present (till the end of April 2021),

India’s installed renewable energy capacity is about 95 GW (more than 50 percent) including 40.5

GW of solar power.

Second statement is incorrect: Southern region has largest installed capacity.

Third statement is correct: You can see below:

Page 38: QUIZ COMPILATIONS – JUNE PART 1

GOALTIDE IAS ACADEMY 37

QUIZ COMPILATIONS – JUNE PART 1

Link: https://science.thewire.in/environment/with-80-gw-to-go-india-looks-set-to-miss-2022-

renewable-energy-target/

Q42. Consider the following statements.

1. India is the largest producer of tea in the world.

2. Nearly 55% of tea produced in India is from Assam.

3. The present Tea Board is functioning as a statutory body of the Central Government under

the Ministry of Commerce.

Which of the above statements is/are correct?

a. 1 and 2 only

b. 2 and 3 only

c. 1 and 3 only

d. 1, 2 and 3

Solution: b

Explanation:

India Tea scenario:

Nearly 55% of tea produced in India is from Assam, with the state supplying 80% of the

country’s exports of the commodity. Second statement is correct.

Page 39: QUIZ COMPILATIONS – JUNE PART 1

GOALTIDE IAS ACADEMY 38

QUIZ COMPILATIONS – JUNE PART 1

Other important data: First statement is incorrect.

Page 40: QUIZ COMPILATIONS – JUNE PART 1

GOALTIDE IAS ACADEMY 39

QUIZ COMPILATIONS – JUNE PART 1

TEA BOARD ORGANISATION AND FUNCTIONS

Organisation of the Board: The present Tea Board is functioning as a statutory body of the

Central Government under the Ministry of Commerce. Third statement is correct.

The Board is constituted of 31 members (including Chairman) drawn from Members of

Parliament, tea producers, tea traders, tea brokers, consumers, and representatives of

Governments from the principal tea producing states, and trade unions. The Board is

reconstituted every three years.

The present Tea Board set up under section 4 of the Tea Act 1953 was constituted on 1st

April 1954.

Q43. Kalbelia dance belongs to:

a. Rajasthan

b. Odisha

c. Maharashtra

d. Chhattisgarh

Solution: a

Explanation:

Kalbeliya performers come from the most marginalized sections of society. A distinguishing feature

of the Kalbeliya community has been its nomadic lifestyle. Earlier, Kalbeliyas moved from one place

to another to secure their livelihood. They have been involved in a range of occupations such as

snake charming, begging, construction work, stone work etc.

Link: https://livewire.thewire.in/art/pandemic-impact-rajasthan-kalbeliya-performers/

Q44. Consider the following statements regarding Performance Grading Index (PGI).

Page 41: QUIZ COMPILATIONS – JUNE PART 1

GOALTIDE IAS ACADEMY 40

QUIZ COMPILATIONS – JUNE PART 1

1. The index is released by NITI Aayog.

2. The purpose of this index is to make primary education more robust, so that a strong

foundation is laid at primary level itself.

Which of the above statements is/are correct?

a. 1 only

b. 2 only

c. Both 1 and 2

d. Neither 1 nor 2

Solution: d

Explanation:

The schemes initiated by the Department of School Education and Literacy (DoSEL) along with the

implementation of the Right of Children to Free and Compulsory Education Act, have resulted in

significant improvement in accessibility. As a logical next step, the focus has now shifted from access

to quality of education. First statement is incorrect.

WHY IT IS REQUIRED? Second statement is also incorrect.

The exercise, which is the first of its kind at such a scale, envisages that the Index will propel the

States and Union Territories (UTs) towards undertaking multi-pronged interventions that will bring

about the much-desired optimal education outcomes. The purpose of the PGI therefore is to help

the States and UTs to pinpoint the gaps and accordingly prioritize areas for intervention to ensure

that the school education system is robust at every level.

The PGI for the States and Union Territories (UTs) was first published in 2019 for the reference year

2017-18. The PGI for reference year 2018-19 was published in the year 2020. The present

publication, PGI 2019 20 at State/UT level, has been prepared with the same set of 70 parameters

used for the two previous PGIs.

Page 42: QUIZ COMPILATIONS – JUNE PART 1

GOALTIDE IAS ACADEMY 41

QUIZ COMPILATIONS – JUNE PART 1

Q45. Consider the following Seas.

Page 43: QUIZ COMPILATIONS – JUNE PART 1

GOALTIDE IAS ACADEMY 42

QUIZ COMPILATIONS – JUNE PART 1

1. Aegean Sea

2. Strait of Gibraltar

3. Adriatic Sea

Arrange the above from East to West.

a. 3-2-1

b. 1-2-3

c. 1-3-2

d. 2-1-3

Solution: c

Explanation:

From now only, you start creating picture of these maps in your mind. So, many times

these images will come in our Practice quiz but with different places. Gradually you will

become expert. See map below.

Q46. According to the constitution of India, which of the following authorities is/are barred

from future appointment to any public offices by the central and state government after

retirement?

1. Finance Commission

2. Chairman of Union Public Service Commission

3. Comptroller and Auditor General of India

Select the correct answer using the code given below.

a. 1 and 2 only

b. 2 and 3 only

c. 1 and 3 only

Page 44: QUIZ COMPILATIONS – JUNE PART 1

GOALTIDE IAS ACADEMY 43

QUIZ COMPILATIONS – JUNE PART 1

d. 1, 2 and 3

Solution: b

Explanation:

Remember here, “AS PER THE CONSTITUTION OF INDIA. These small things can help you in

exam.

Chairman of UPSC is barred from further appointment by government after his

retirement.

CAG is also not eligible for re appointment.

FINANCE COMMISSION ARE NOT BARRED. THE CAN BE RE-APPOINTED. Read a small story

below:

The Constitution has not mentioned about reappointment of Finance Commission.

Parliament by law, but law enacted by it mentions that Finance Commission can be re-

appointed.

Page 45: QUIZ COMPILATIONS – JUNE PART 1

GOALTIDE IAS ACADEMY 44

QUIZ COMPILATIONS – JUNE PART 1

Q47. Consider the following statements regarding Treasury Bills.

1. They are the short- term debt instruments issued by the Government of India.

2. Interest rate is decided by the government only.

Which of the above statements is/are correct?

a. 1 only

b. 2 only

c. Both 1 and 2

d. Neither 1 nor 2

Solution: a

Explanation:

Treasury bills or T-bills, which are money market instruments, are short term debt

instruments issued by the Government of India and are presently issued in three tenors,

namely, 91-day, 182 day and 364 days.

Treasury bills are zero coupon securities and pay no interest. Instead, they are issued at a

discount and redeemed at the face value at maturity. For example, a 91-day Treasury bill of

₹100/- (face value) may be issued at say ₹ 98.20, that is, at a discount of say, ₹1.80 and

would be redeemed at the face value of ₹100/-. Interest rates are not used here. Therefore,

second statement is wrong.

Q48. Consider the following statements regarding Biomining.

1. It is the process of using microorganisms (microbes) to extract metals of economic

interest from rock ores or mine waste.

2. The use of this technique is limited to metals which can be oxidized by the microbes.

Page 46: QUIZ COMPILATIONS – JUNE PART 1

GOALTIDE IAS ACADEMY 45

QUIZ COMPILATIONS – JUNE PART 1

3. This technique is not suitable for metals like iron and copper, which are dissolved

easily in water, for such minerals other techniques are used.

Which of the above statements is/are correct?

a. 1 and 2 only

b. 1 and 3 only

c. 1 only

d. 2 and 3 only

Solution: c

Explanation:

News is:

Biomining is the process of using microorganisms (microbes) to extract metals of economic

interest from rock ores or mine waste. Biomining techniques may also be used to clean up

sites that have been polluted with metals.

Page 47: QUIZ COMPILATIONS – JUNE PART 1

GOALTIDE IAS ACADEMY 46

QUIZ COMPILATIONS – JUNE PART 1

Q49. Consider the following Committees of the Lok Sabha.

1. Committee on Private Members’ Bills and Resolutions.

2. Rules Committee

3. General Purposes Committee

Which of the above committees does Speaker has its ex-officio chairman?

a. 1 only

b. 2 only

c. 2 and 3 only

d. 1, 2 and 3

Solution: c

Explanation:

Q50. Proper design and effective implementation of UN-REDD+ Programme can significantly

contribute to:

1. protection of biodiversity

Page 48: QUIZ COMPILATIONS – JUNE PART 1

GOALTIDE IAS ACADEMY 47

QUIZ COMPILATIONS – JUNE PART 1

2. resilience of forest ecosystems

3. poverty reduction

Select the correct answer using the code given below.

a. 1 and 2 only

b. 3 only

c. 2 and 3 only

d. 1, 2 and 3

Solution: d

Explanation:

First two statements, you should have no doubt.

Poverty reduction is nowhere mentioned in the REDD+. But the question is not asking for

specific details. It is like, “UN-REDD+ Programme can significantly contribute to?”

Conservation, sustainable management of forests and enhancement of forest carbon stocks

in developing countries will certainly contribute to employment opportunities and help in

poverty reduction.

FAO also says the same “REDD+ can also contribute to achieving other SDGs – including

those which address poverty reduction, health and well-being, hunger alleviation, and

improving institutions”.

Page 49: QUIZ COMPILATIONS – JUNE PART 1

GOALTIDE IAS ACADEMY 48

QUIZ COMPILATIONS – JUNE PART 1

Q51. Consider the following Pairs.

Communities in news Belongs to

1. Tongias North Eastern region of India

2. Van Gujjars Himalayan states like Jammu and Kashmir, Himachal Pradesh and Uttarakhand.

Which of the above pairs is/are correct?

a. 1 only

b. 2 only

c. Both 1 and 2

d. Neither 1 nor 2

Solution: b

Explanation:

Communities in news Belongs to

Tongias Shivalik hills in Uttarakhand and Uttar Pradesh.

Van Gujjars Himalayan states like Jammu and Kashmir, Himachal Pradesh and Uttarakhand.

Link: https://india.mongabay.com/2021/06/shivaliks-tongia-and-van-gujjar-communities-protect-

wildlife-but-their-rights-are-ignored/

Q52. Which of the following can be predominantly found in Lakshadweep Island?

1. Sea cucumbers

2. Corals Atolls

3. Leatherback turtles

Select the correct code.

a. 1 and 2 only

b. 2 and 3 only

c. 1 and 3 only

d. 1, 2 and 3

Solution: a

Explanation:

Leatherback turtles predominantly can be observed in Andaman and Nicobar. Very rarely it is seen

in Lakshadweep.

News was also there:

Page 50: QUIZ COMPILATIONS – JUNE PART 1

GOALTIDE IAS ACADEMY 49

QUIZ COMPILATIONS – JUNE PART 1

Q53. Consider the following statements.

1. The Food Corporation of India (FCI) was established under National Food Security Act, 2013.

2. FCI also facilitates sale of surplus stock under Open Market Sales Scheme.

3. Movement and distribution of the food grains is important function of FCI.

4. The Government of India announces the Minimum Support Prices (MSP) as per the

recommendation of FCI.

Which of the above statements above is/are incorrect?

Select the correct code.

a. 2 and 4 only

b. 1 and 4 only

c. 4 only

d. 2 and 3 only

Solution: b

Explanation:

First statement is incorrect.

The Food Corporation of India (FCI) was established under Food Corporations Act, 1964.

Second statement is correct.

FCI also facilitates sale of surplus stock under Open Market Sales Scheme. This is done to supply the

markets with food grains, especially during the lean season and to avert a situation of price rise due

to lower supply of the food grains. FCI is supposed to transfer food grains to the deficit regions for

this purpose.

Third statement is also correct:

Page 51: QUIZ COMPILATIONS – JUNE PART 1

GOALTIDE IAS ACADEMY 50

QUIZ COMPILATIONS – JUNE PART 1

Movement and distribution of the food grains is another important function of FCI. This is to

ensure that the food grains are distributed to states as per the requirement of various welfare

measures announced by the government.

FCI undertakes movement for the following purposes:

• To evacuate food grain stocks from the surplus regions.

• Supply them to deficit regions for distribution through PDS and other schemes.

• To create buffer stocks in deficit region.

Fourth Statement is incorrect.

The Government of India announces the Minimum Support Prices (MSP) as per the recommendation

of Commission of Agricultural Costs and Prices (CACP), prior to every harvest (Rabi/kharif season).

Q54. Consider the following statements regarding “Raising and Accelerating MSME Productivity”

(RAMP) initiative.

1. The program will be assisted by World Bank.

2. It aims to address the immediate liquidity and credit needs of millions of viable MSMEs

severely impacted by the ongoing COVID-19 pandemic.

Which of the above statements is/are correct?

a. 1 only

b. 2 only

c. Both 1 and 2

d. Neither 1 nor 2

Solution: c

Explanation:

“Raising and Accelerating MSME Productivity” (RAMP) is a five-year duration (2021-22 to 2025-26)

new Central Sector Scheme, with a total project cost of INR 6062.45 Cr, of which the World Bank

contribution is INR 3750 Cr (USD 500 Mn) and the remaining is funded by Govt. of India.

Page 52: QUIZ COMPILATIONS – JUNE PART 1

GOALTIDE IAS ACADEMY 51

QUIZ COMPILATIONS – JUNE PART 1

It is the World Bank’s second intervention in this sector, the first being the $750 million MSME

Emergency Response Program, approved in July 2020 to address the immediate liquidity and credit

needs of millions of viable MSMEs severely impacted by the ongoing COVID-19 pandemic.

Having supported the immediate liquidity and credit needs of viable MSMEs in the first phase, the

RAMP Program will support the Government of India’s efforts to increase MSME productivity and

financing in the economic recovery phase, crowd in private sector financing in the medium term,

and tackle long-standing financial sector issues that are holding back the growth of the MSME

sector.

A. There is a need for “convergence” of policies, programs, and schemes at all levels. To bring

about this fundamental shift, the program will help setup a high-level MSME Council to enable

better coordination between national and state-level programs.

B. The RAMP program will provide better access to finance and working capital for MSMEs by

strengthening the receivable financing markets; and scale-up online dispute resolution

mechanisms to address the problem of delayed payments.

C. The program will promote technology-based solutions, green investments, and access to

services for women-headed businesses. It will also build partnerships with the private sector as

service providers to reach greater scale.

D. In addition to national level activities, the program will initiate targeted activities in five “first

mover” states – Gujarat, Maharashtra, Punjab, Rajasthan, and Tamil Nadu with the potential of

additional states joining the Program going forward.

Link: Link: https://www.worldbank.org/en/news/press-release/2021/06/04/world-bank-approves-

500-million-program-to-strengthen-performance-of-micro-small-and-medium-enterprises-in-india

Q55. Consider the following countries.

1. Croatia

2. Serbia

3. Luxembourg

4. Macedonia

Which of the above countries of Europe is/are land locked?

a. 1 and 2 only

b. 2, 3 and 4 only

c. 3 and 4 only

d. 1 and 3 only

Solution: b

Explanation:

Croatia is not a land-locked nation.

Page 53: QUIZ COMPILATIONS – JUNE PART 1

GOALTIDE IAS ACADEMY 52

QUIZ COMPILATIONS – JUNE PART 1

Q56. Consider the following statements regarding “National Commission of Minorities

(NCM)”.

1. With the enactment of the National Commission for Minorities Act, 1992, the

Minorities Commission became a statutory body.

2. Under Section of the NCM Act, it can declare new minority communities.

Which of the above statements is/are correct?

a. 1 only

b. 2 only

c. Both 1 and 2

d. Neither 1 nor 2

Solution: a

Explanation:

Page 54: QUIZ COMPILATIONS – JUNE PART 1

GOALTIDE IAS ACADEMY 53

QUIZ COMPILATIONS – JUNE PART 1

The role of the NCM was not to declare new minority communities. The power to declare a

community as minority lies with the Central government. Section 2(c) of the NCM Act itself

clearly states that a community is notified as “minority” only by the government.

It consists of a Chairperson, a Vice Chairperson and five Members to be nominated by the

Central Government from amongst persons of eminence, ability and integrity.

Page 55: QUIZ COMPILATIONS – JUNE PART 1

GOALTIDE IAS ACADEMY 54

QUIZ COMPILATIONS – JUNE PART 1

Q57. Vaibhashika and Sautrantika belongs to which sect?

a. Jainism

b. Buddhism

c. Vaishnavism

d. Shaivism

Solution: b

Explanation:

Hinayana was later divided into two sects i.e., Vaibhashika and Sautrantika.

Q58. Which is/are the constitutionally mandated bodies under the 74th Constitutional

Amendment Act, 1992?

1. State Finance Commission.

2. State Election Commission.

3. District Planning Committee.

4. Metropolitan Planning Committee

Choose the correct code from the below.

Page 56: QUIZ COMPILATIONS – JUNE PART 1

GOALTIDE IAS ACADEMY 55

QUIZ COMPILATIONS – JUNE PART 1

a. 2 and 4 only

b. 1, 2 and 3 only

c. 2, 3 and 4 only

d. 1, 2, 3 and 4

Solution: d

Explanation:

Q59. Which among are the reasons for biodiversity loss?

1. Environmental pollution

2. introduction of exotic species

3. over-exploitation of resources

4. Fragmentation of habitat

Select the correct answers using the codes given below

(a) 1 and 4 only

(b) 1, 2 and 3 only

(c) 3 and 4 only

(d) All of the above

Page 57: QUIZ COMPILATIONS – JUNE PART 1

GOALTIDE IAS ACADEMY 56

QUIZ COMPILATIONS – JUNE PART 1

Answer: d

Explanation:

Habitat fragmentation will lead to the habitat destruction. For example, construction of

roads, railway line will fragment the habitat. This will reduce food availability and reduced

reproduction. This will ultimately reduce the biodiversity.

Q60. ‘NOTA’ option is available to voters in which of the following elections:

1. Lok Sabha elections

2. Rajya Sabha elections

3. State Assembly elections

4. State Council elections

Select the correct Answer using the codes given below:

a. 1 and 3 only

b. 2 and 4 only

c. 1 only

d. 1, 2, 3, and 4

Solution: a

Explanation:

''NOTA'' or ''none of the above'' is a ballot option provided in the elections to Indian voters.

Through NOTA, a citizen has the right to not vote for any candidate contesting the elections.

NOTA is an option is direct elections like Lok Sabha and State Assemblies. Whereas elections

to Rajya Sabha and State Councils are Indirect elections. Hence Option ‘B’ and ‘D’ can’t be

the Answer.

Option C has only Lok Sabha, so this is also eliminated.

Q61. Consider the following statements.

1. The President appoints both Chief Election Commissioner and Election Commissioners.

2. If the Chief Election Commissioner and other Election Commissioners differ in opinion

on any matter, such matter shall be decided according to the opinion of the majority.

3. The Tenure of members of Election Commission is not provided by the Constitution.

4. Election Commissioners and Chief Election Commissioner shall not be removed from

his office except in like manner and on the like grounds as a Judge of the Supreme

Court.

Which of the above statements is/are correct?

a. 1 and 2 only

b. 1, 2 and 3 only

c. 2, 3 and 4 only

d. 1 and 4 only

Page 58: QUIZ COMPILATIONS – JUNE PART 1

GOALTIDE IAS ACADEMY 57

QUIZ COMPILATIONS – JUNE PART 1

Solution: b

Explanation:

First statement is correct:

The President appoints Chief Election Commissioner and Election Commissioners.

Second statement is correct: Provided under Election Commission (Conditions of Service

of Election Commissioners and Transaction of Business) Act, 1991.

Third statement is also correct: So, where it is provided? It is also provided under same Act

as mentioned above: Election Commission (Conditions of Service of Election

Commissioners and Transaction of Business) Act, 1991.

Fourth statement is incorrect: Only Chief Election Commissioner is removed in said

manner.

Q62. Consider the following statements.

1. International Criminal Court is an agency of United Nations established to investigate

individuals accused of crimes of genocide, war crimes, etc.

2. India has neither signed nor ratified the Rome Statute of the International Criminal

Court (ICC).

Which of the above statements is/are correct?

Page 59: QUIZ COMPILATIONS – JUNE PART 1

GOALTIDE IAS ACADEMY 58

QUIZ COMPILATIONS – JUNE PART 1

a. 1 only

b. 2 only

c. Both 1 and 2

d. Neither 1 nor 2

Solution: b

Explanations:

THE FIRST STATEMENT WILL CREATE PROBLEM ONLY IF YOU DON’T KNOW IT IS UN AGENCY

OR NOT.

The International Criminal Court (ICC) investigates and, where warranted, tries individuals

charged with the gravest crimes of concern to the international community: genocide, war

crimes, crimes against humanity and the crime of aggression.

India has neither signed nor ratified the Rome Statute on the International Criminal Court

(ICC). Why India has not signed?? Read below don’t think now.

India considers the inherent jurisdiction of the ICC as a violation of a nation’s sovereignty.

Therefore, India has insisted for having an ‘opt-in’ provision whereby a country could accept

the jurisdiction of the ICC by declaration, specified to an issue and time period.

India is resistant to accepting the inherent jurisdiction of the ICC as it would be seen as

being superior to Indian judicial system.

Page 60: QUIZ COMPILATIONS – JUNE PART 1

GOALTIDE IAS ACADEMY 59

QUIZ COMPILATIONS – JUNE PART 1

India remains hesitant towards the ICC as it can be used with political motives against India

with regard to Kashmir and in other matters of India’s internal affairs.

Q63. Consider the following statements:

1. The term ‘Union of India’ includes only States while the term ‘Territory of India’

includes not only the States but also the Union Territories.

2. The Indian Constitution provides for the admission or establishment of new States

that are not part of the Union of India by amending the Constitution under Article

368.

Which of the statements given above is/are correct?

a. 1 only

b. 2 only

c. Both 1 and 2

d. Neither 1 nor 2

Solution: a

Explanation:

Statement 1 is correct:

The ‘Territory of India’ is a wider expression than the ‘Union of India’ because the latter

includes only States while the former includes not only the States but also Union

Territories and territories that may be acquired by the Government of India at any future

time.

The States are the members of the federal system and share distribution of powers with the

Centre. The Union Territories and the acquired territories, on the other hand, are directly

administered by the Central government. Being a sovereign state, India can acquire foreign

territories according to the modes recognised by international law, i.e., cession (following

treaty, purchase, gift, lease or plebiscite), occupation (hitherto unoccupied by a recognised

ruler), conquest or subjugation.

Page 61: QUIZ COMPILATIONS – JUNE PART 1

GOALTIDE IAS ACADEMY 60

QUIZ COMPILATIONS – JUNE PART 1

Statement 2 is incorrect: Article 2 empowers the Parliament to ‘admit into the Union of

India, or establish, new States on such terms and conditions as it thinks fit’. Thus, Article 2

grants two powers to the Parliament:

a. the power to admit into the Union of India new States; and

b. The power to establish new States.

Article 2 relates to the admission or establishment of new States that are not part of the

Union of India. Article 3, on the other hand, relates to the formation of or changes in the

existing States of the Union of India. In other words, Article 3 deals with the internal re-

adjustment inter se of the territories of the constituent States of the Union of India.

ARTICLE 4 SAYS: Any change under Article 2 and 3 shall not be called as amendment under

Article 368.

Q64. Consider the following bodies.

1. Municipalities

2. Rajya Sabha

3. Legislative Council

4. Lok Sabha

Which of the above bodies has/have the provision of reservation of seats for SC/ST?

a. 2, 3 and 4 only

b. 4 only

c. 1, 3 and 4 only

d. 1 and 4 only

Solution: d:

Explanation:

Article 334 provides that seats should be reserved for SC/STs in Lok Sabha and Vidhan

Sabhas. This same article also ensures that Anglo-Indians too get representation in these

bodies.

There’s no reservation for Rajya Sabha and State Legislative Council.

In Panchayats and Municipalities, there is a provision for reservation of SC/ST. See below

for Municipalities.

Page 62: QUIZ COMPILATIONS – JUNE PART 1

GOALTIDE IAS ACADEMY 61

QUIZ COMPILATIONS – JUNE PART 1

Q65. In the context of India, consider the following statements.

1. Bonded Labour System in India is regulated by Bonded Labour System (Abolition)

Act, 1976.

2. Under the Constitution of India, Labour is a subject in the State List.

3. Article 24 prohibits trafficking in forced labor.

Which of the above statements is/are correct?

a. 1 and 2 only

b. 3 only

c. 1 and 3 only

d. 1 only

Solution: d

Explanation:

Article 23 of the 1949 Constitution of India outlaws both the trafficking of human beings

and forced labor, but the legislation defining and banning bonded labor was only approved

by Parliament in 1976. See below.

The issue of 'bonded labour' came to the list of national priority when it was included in the

old 20-Point Programme in 1975. The Bonded Labour System (Abolition) Ordinance was

promulgated on 25th October 1975. This was later on replaced by the Bonded Labour

System (Abolition) Act, 1976.

Also, under the Constitution of India, Labour is a subject in the Concurrent List where both

the Central & State Governments are competent to enact legislation subject to certain

matters being reserved for the Centre.

Page 63: QUIZ COMPILATIONS – JUNE PART 1

GOALTIDE IAS ACADEMY 62

QUIZ COMPILATIONS – JUNE PART 1

Q66. The ‘Red Data Books’ published by the International Union for Conservation of Nature

and Natural Resources (IUCN) contain lists of

1. Endemic plant and animal species present in the biodiversity hotspots.

2. Threatened plant and animal species.

3. Protected sites for conservation of nature & natural resources in various countries.

Which of the statement given above is/are correct?

a. 1 and 2 only

b. 2 only

c. 2 and 3 only

d. 1, 2 and 3

Solution: b

Explanation:

It publishes Red data book which contains a list of ‘Threatened species’ (vulnerable,

endangered and critically endangered).

Q67. Which of the following are Solar Geoengineering techniques to reduce Global

Warming?

1. Aerosol injection

2. Marine cloud brightening

3. High-albedo crops and buildings

4. Cloud thinning

Select the correct code.

a. 1, 2 and 3 only

b. 2, 3 and 4 only

c. 1, 3 and 4 only

d. 1, 2, 3 and 4

Solution: d

Explanation:

Nothing in this question. Just need to understand one term Solar Geoengineering and you

will solve this question easily.

Solar geoengineering is a term used to describe a group of hypothetical technologies that

could, in theory, counteract temperature rise by reflecting more sunlight away from the

Earth’s surface.

Page 64: QUIZ COMPILATIONS – JUNE PART 1

GOALTIDE IAS ACADEMY 63

QUIZ COMPILATIONS – JUNE PART 1

Page 65: QUIZ COMPILATIONS – JUNE PART 1

GOALTIDE IAS ACADEMY 64

QUIZ COMPILATIONS – JUNE PART 1

Q68. Which one of the following is NOT the most likely measure the Government/RBI takes

to control rupee weakening?

A. Increasing tariffs on imported goods and promoting exports

B. Easing Foreign Direct Investment Rules

C. Increase in the issue of rupee dominated Masala Bonds

D. Following an expansionary monetary policy

Solution: d

Explanation:

Rupee depreciation means that Indian rupee is losing its value against foreign currencies.

In simple terms, you will remember:

Rupee depreciates when more rupee in market compared to dollar. More rupee in market

means, its value will decrease. So, to prevent depreciation, we have to keep dollars in our

economy.

When you see in the papers that rupee has become stronger/weaker or has gained some

points or lost them, it simply means the value of rupee against dollar has

increased/decreased. e.g., Currently $1=67.25 rupees. If it says rupee gained/became

stronger than $1=67/66/65. you get the point. When it says gained, the numbers actually go

down for rupee. It is so because now we have to spend less to buy a dollar so we are the

gainers. Thus, it enables us to buy more goods from the commodity basket (a set of

consumer goods and services offered on the consumer market e.g., Personal care goods,

household equipments.)

Now back to question.

First statement is correct. It will reduce CAD. Less dollar will now go outside.

Second statement is correct.

The FDIs are driven in the form of purchase of government bonds and corporate bonds.

Foreign investors buy these bonds by shelling out foreign currency. Thus, the value of the

foreign currency with respect to the Indian Rupee drops as the demand for Indian rupee has

Page 66: QUIZ COMPILATIONS – JUNE PART 1

GOALTIDE IAS ACADEMY 65

QUIZ COMPILATIONS – JUNE PART 1

increased against the Dollar. This increase in demand for INR causes the rupee to

appreciate against the Dollar.

Thus, FDIs certainly help any country (India is no exception) to appreciate its currency.

Third statement is correct.

It will help by reducing the demand of dollars in loan-repayment.

Fourth statement is wrong.

because rupee currency supply will increase without corresponding increase in the supply

of dollars and as a result: dollar will strengthen, Indian Rupee will weaken further. So D is

the answer.

Q69. With reference to Asian Development Bank (ADB), consider the following statements:

1. ADB was formed after 1991 financial crisis to help developing nations of Asia and

Pacific region.

2. India is the largest shareholder in AIB in Asia-Pacific region.

3. More than 50 percent members are from within Asia and the Pacific.

Which of the statements given above is/are correct?

a. 1 and 2 only

b. 2 and 3 only

c. 3 only

d. 1 and 3 only

Solution: c

Explanation:

First statement is incorrect.

Second statement is incorrect: Japan is the largest shareholder.

Page 67: QUIZ COMPILATIONS – JUNE PART 1

GOALTIDE IAS ACADEMY 66

QUIZ COMPILATIONS – JUNE PART 1

Third statement is correct.

As of 31 December 2018, ADB's shareholders consist of 48 developing and developed

members within Asia and the Pacific region, and 19 members from outside the region.

Q70. With reference to the Mathura School of sculpture, consider the following statements:

1. It was influenced by Buddhism alone.

2. The sculptures were made using spotted red sandstone.

3. It was patronized by the Kushanas.

Which of the statements given above is/are correct?

a. 2 and 3 only

b. 1 and 3 only

c. 3 only

d. 1, 2 and 3

Solution: a

Explanation:

Mathura School of Art flourished mainly during the reign of Kushana emperor Kanishka.

Mathura School had developed indigenously. The Mathura school images include those of

Buddha, Bodhisattvas, Vishnu, Shiva, Yakshas, Yakshini, Jinas etc. representing religious zeal

of Brahmanism, Jainism and Buddhism. So, statement (1) is not correct.

Page 68: QUIZ COMPILATIONS – JUNE PART 1

GOALTIDE IAS ACADEMY 67

QUIZ COMPILATIONS – JUNE PART 1

The ingredients used in sculptures of Mathura school were made up of spotted red

sandstone. So, statement (2) is correct.

During the first century AD, Gandhara and Mathura school of Art flourished mainly during

the reign of Kushana emperor Kanishka. So, statement (3) is correct.

Hence, the correct answer is a.

Q71. Arrange the following from North to South.

1. Dhaka

2. Hanoi

3. Manila

4. Singapore

Select the correct code.

a. 1-2-3-4

b. 2-1-3-4

c. 1-3-2-4

d. 3-1-2-4

Solution: a

Explanation:

Page 69: QUIZ COMPILATIONS – JUNE PART 1

GOALTIDE IAS ACADEMY 68

QUIZ COMPILATIONS – JUNE PART 1

Q72. Who among the following has the power to request a retired Supreme Court Judge to

act as a Judge of the Supreme Court for a temporary period?

a. President of India with the previous consent of the Chief Justice of India.

b. Chief Justice of India with the previous consent of the President.

c. It is the power of the collegium to suggest a name to the President.

d. There is no such provision in the Constitution of India

Solution: b

Explanation:

As per provisions of the Article 128, Chief Justice of India, with the previous consent of the

President, request a retired Judge of the Supreme Court High Court, who is duly qualified for

appointment as a Judge of the Supreme Court, to sit and act as a Judge of the Supreme

Court.

Page 70: QUIZ COMPILATIONS – JUNE PART 1

GOALTIDE IAS ACADEMY 69

QUIZ COMPILATIONS – JUNE PART 1

Q73. In which of the following parts of the Constitution, liberty of faith and worship has

been given to the people of India?

a. Preamble.

b. Preamble and Fundamental Rights

c. Preamble, Fundamental Rights and Directive Principles.

d. Preamble, Directive Principles and Fundamental Duties.

Solution: b

Explanation:

The fundamentals of the Indian Constitution are contained in the Preamble which secures

its citizens, Justice, social, economic and political, liberty of thought, expression, belief, faith

and worship. Also, Article 25 to 28 of the Constitution of India guarantees the right of

freedom of religion to its people.

Q74. Consider the following macro-economic tools:

1. Capital Expenditure

2. Monetary Policy

3. Fiscal Policy

4. Revenue Expenditure

An income guarantee scheme for the citizens through a direct transfer of money to their

bank accounts is an example of which of the above macroeconomic measures?

a. 3 and 4 only

b. 1 and 3 only

c. 2 and 3 only

d. 2 and 4 only

Solution: a

Explanations:

Any income guarantee scheme like, Pradhan Mantri Kisan Samman Nidhi (giving 6000 Rs

every year to farmers) or Universal Basic Income is an example of the fiscal policy.

This is because it is a subsidy (a transfer payment from the government to its people) and

such government expenditures are part of fiscal policy.

Revenue expenditure is an expenditure incurred by the Government on its departments,

various services, interest on debt, subsidies, etc. Broadly speaking, expenditure that does

not result in creation of assets is treated as revenue expenditure. As it is a form of subsidy it

forms part of revenue expenditures.

Page 71: QUIZ COMPILATIONS – JUNE PART 1

GOALTIDE IAS ACADEMY 70

QUIZ COMPILATIONS – JUNE PART 1

The income guarantee scheme does not come under capital expenditure as it does not

create any assets immediately.

Monetary policy refers to control and regulation of money supply. It is primarily the job of

the Reserve Bank of India (RBI).

Q75. With reference to the National Anti-Doping Agency (NADA), consider the following

statements:

1. The primary objectives are to implement anti-doping rules as per WADA code and

regulate dope control programme.

2. Union Minister is the chairman of the governing body of NADA.

Which of the statements given above is/are correct?

a. 1 only

b. 2 only

c. Both 1 and 2

d. Neither 1 nor 2

Solution: c

Explanation:

Understand NADA:

National Anti-Doping Agency (NADA) was set up as registered society under the Societies

Registration Act of 1860 on November 24, 2005 with a mandate for Dope free sports in

India. The primary objectives are to implement anti-doping rules as per WADA code,

regulate dope control programme, to promote education and research and creating

awareness about doping and its ill effects.

There is a Governing Body of NADA. The Chairman of this Governing Body is Union Sports

Minister.

Page 72: QUIZ COMPILATIONS – JUNE PART 1

GOALTIDE IAS ACADEMY 71

QUIZ COMPILATIONS – JUNE PART 1

Q76. Which of the following would be the consequences if the magnetic field of Earth is

flipped?

1. The life at the surface will be exposed to higher amounts of solar radiation.

2. It can damage satellites, cause power outages and interrupt radio communications

3. It could lead to the formation of ozone holes.

Select the correct answer from the codes given below:

a. 1 only

b. 1 and 2 only

c. 2 and 3 only

d. 1, 2 and 3

Solution: d

Explanation:

Introduction to topic:

The geologic record shows that hundreds of pole reversals have occurred throughout

Earth's history; they happen when patches of iron atoms in Earth's liquid outer core become

reverse-aligned, like tiny magnets oriented in the opposite direction from those around

them. When the reversed patches grow to the point that they dominate the rest of the core,

Earth's overall magnetic field flips. The last reversal happened 780,000 years ago during the

Stone Age, and indeed there's evidence to suggest the planet may be in the early stages of a

pole reversal right now.

Page 73: QUIZ COMPILATIONS – JUNE PART 1

GOALTIDE IAS ACADEMY 72

QUIZ COMPILATIONS – JUNE PART 1

The most dramatic changes that occur when the poles reverse is a very large decrease of

the total field intensity. It's not a sudden flip, but a slow process, during which the field

strength becomes weak.

So, what happens when field strength becomes weak?

A strong magnetic field helps protect Earth from blasts of radiation from the sun. Coronal

mass ejections (CMEs) occasionally occur on the Sun, and sometimes hurtle directly toward

Earth. With a weak field, this shielding is less efficient. Flipping any magnetic field ensures

earth being highly exposed to higher amounts of solar radiation.

Ozone formation:

The charged particles bombarding Earth's atmosphere during solar storms would punch

holes in Earth's atmosphere, and this could hurt humans. Ozone holes, like that over

Antarctica (which today are due to an entirely different cause related to man) could form as

solar particles interact with the atmosphere in a cascade of chemical reactions.

Disrupt technologies

Our technology definitely would be in danger, however. Even now, solar storms can

damage satellites, cause power outages and interrupt radio communications. "These kinds

Page 74: QUIZ COMPILATIONS – JUNE PART 1

GOALTIDE IAS ACADEMY 73

QUIZ COMPILATIONS – JUNE PART 1

of negative influences clearly will increase if the magnetic field and thus its shielding

function became significantly weaker.

Effect on species

One additional worry is that a weakening and eventual reversal in the field would disorient

all those species that rely on geomagnetism for navigation, including bees, salmon, turtles,

whales, bacteria and pigeons. There is no scientific consensus on how those creatures would

cope.

Q77. In the context of the Reserve Bank of India, what is the meaning of Ways and Means

Advances (WMA)?

a. Financial aid to debt-ridden banks

b. Loan facility to both the Central and the State governments to meet their cash

requirements

c. Advanced payments to entrepreneurs under Make in India

d. Loans which are given to foreign governments

Solution: b

Explanation:

The Reserve Bank of India acts as the banker to both the Central and State Governments.

The government used to borrow money from the RBI through treasury bills, but Ways and

Means Advances replaced it in 1997. It acts as a loan facility to both Central and State

governments to meet their cash requirements. The loans taken by the government through

Ways and Means Advances need to be returned in 90 days. The interest rate of WMA is

currently the repo rate, and its limits are decided mutually by the RBI and the Government

of India.

Q78. After 1991, Indian foreign exchange reserves have rapidly increased due to various

reasons:

1. Revaluation of rupee

2. Availability of foreign exchange from NRIs under various investment

3. Increased foreign investment

4. Full convertibility of Rupee on the current account

Select the correct code.

a. 1 and 2 only

b. 1 and 4 only

c. 2, 3 and 4 only

d. 1, 2, 3 and 4

Solution: c

Page 75: QUIZ COMPILATIONS – JUNE PART 1

GOALTIDE IAS ACADEMY 74

QUIZ COMPILATIONS – JUNE PART 1

Explanation:

These questions should never be wrong.

Easily you can understand. No need much explanation.

The rapid increase of forex reserve after 1991 was because of many factors including

availability of foreign exchange from NRIs under various investment, increased capital

inflow and full convertibility of rupee on the current account.

Indian foreign exchange reserves have rapidly increased due to devaluation of the rupee.

Devaluing a currency reduces the cost of a country’s exports and hence gains foreign

exchange for the country.

Q79. With respect to the agricultural technology, consider the following statements:

1. Archaeological evidence for the use of 'ironless' plough is found from Kalibangan, an

Indus valley culture site.

2. Miftah-ul Fuzala talks about the use of a plough with an iron share drawn by two

yoked oxen.

Which of the statements given above is/are correct?

a. 1 only

b. 2 only

c. Both 1 and 2

d. Neither 1 nor 2

Solution: c

Explanation:

Page 76: QUIZ COMPILATIONS – JUNE PART 1

GOALTIDE IAS ACADEMY 75

QUIZ COMPILATIONS – JUNE PART 1

Q80. Concerning Indian history, who among the following described as “Buddha of the Present”? a. Shakyamuni b. Rishabhdeva c. Maitreya d. Dipankara Solution: a Explanation:

Page 77: QUIZ COMPILATIONS – JUNE PART 1

GOALTIDE IAS ACADEMY 76

QUIZ COMPILATIONS – JUNE PART 1

Q81. Consider the following statements.

1. With USD 600 billion mark, India has become second largest reserve holder in the

world after US.

2. SDRs are the largest contributor to reserves after foreign currency assets.

3. If the value of the Rupee decreases due to an increase in demand of the foreign

currency, then RBI buys the dollar in the Indian money market so that depreciation

of the Indian currency can be checked.

Which of the above statements is/are incorrect?

a. 1 only

b. 1 and 2 only

c. 2 and 3 only

d. 1, 2 and 3

Solution: d

Explanation:

First statement is incorrect:

India's foreign exchange reserves crossed $600 for the first time. As on June 4, the foreign

exchange reserves stood at $605 billion, almost tying with Russia as the fourth largest

reserve holder in the world. Not second.

While India’s forex reserve was $605.008 billion, Russia’s was $605.2 billion.

Page 78: QUIZ COMPILATIONS – JUNE PART 1

GOALTIDE IAS ACADEMY 77

QUIZ COMPILATIONS – JUNE PART 1

Second statement is incorrect:

Purpose of the Foreign Exchange Reserve: Third statement is incorrect:

The most significant objective behind this is to ensure that RBI has backup funds if their

national currency rapidly devalues or becomes altogether insolvent.

If the value of the Rupee decreases due to an increase in demand of the foreign currency

then RBI sells the dollar in the Indian money market so that depreciation of the Indian

currency can be checked.

Link: https://www.business-standard.com/article/economy-policy/at-605-bn-india-ties-

with-russia-as-fourth-largest-forex-reserves-holder-121061101316_1.html

Q82. Which of the following programmes is/are set up under NITI Aayog?

1. “Raising and Accelerating MSME Productivity” (RAMP)

2. SAGE (Seniorcare Ageing Growth Engine) initiative

3. Atal innovation mission

Select the correct code.

a. 1 and 2 only

b. 3 only

c. 2 and 3 only

d. 1 and 3 only

Solution: b

Explanation:

Everyone knows Atal Innovation Mission. It has also been asked in Prelims 2019.

Page 79: QUIZ COMPILATIONS – JUNE PART 1

GOALTIDE IAS ACADEMY 78

QUIZ COMPILATIONS – JUNE PART 1

“Raising and Accelerating MSME Productivity” (RAMP) is a five-year duration (2021-22 to

2025-26) new Central Sector Scheme, with a total project cost of INR 6062.45 Cr, of which

the World Bank contribution is INR 3750 Cr (USD 500 Mn) and the remaining is funded by

Govt. of India.

It aims to address the immediate liquidity and credit needs of millions of viable MSMEs

severely impacted by the ongoing COVID-19 pandemic.

SAGE Project is under Ministry of Social Justice and Empowerment.

Q83. Consider the following statements regarding Bay of Bengal Initiative for Multi-Sectoral

Technical and Economic Cooperation (BIMSTEC).

1. India recently hosted 14th BIMSTEC Virtual Summit in 2021.

2. All members of BIMSTEC are from South Asia.

3. Its Secretariat is in New Delhi, India.

Which of the above statements is/are incorrect?

a. 1 and 3 only

b. 2 only

c. 2 and 3 only

d. 1, 2 and 3

Solution: d

Explanation:

First statement is incorrect:

Here, number was not important. Important was, you should know that since its inception

in 2004, only four summits have been held.

Page 80: QUIZ COMPILATIONS – JUNE PART 1

GOALTIDE IAS ACADEMY 79

QUIZ COMPILATIONS – JUNE PART 1

BIMSTEC Summit: Only four BIMSTEC Summits till now. This is the highest policy making

body in the BIMSTEC process.

Second statement is incorrect: It constitutes seven Member States: five deriving from South

Asia, including Bangladesh, Bhutan, India, Nepal, Sri Lanka, and two from Southeast Asia,

including Myanmar and Thailand.

Third statement is incorrect:

Page 81: QUIZ COMPILATIONS – JUNE PART 1

GOALTIDE IAS ACADEMY 80

QUIZ COMPILATIONS – JUNE PART 1

BIMSTEC Secretariat

After a span of 17 years of the founding of BIMSTEC as a regional organization, it’s long

cherished Permanent Secretariat was established in Dhaka, Bangladesh on 13th

September 2014 to serve the BIMSTEC Member States.

Q84. Consider the following statements regarding All India Survey on Higher Education

(AISHE) 2019-20.

1. It is released by NITI Aayog.

2. Total Enrolment in Higher Education has increased.

3. Gender Parity Index (GPI) has improved.

Which of the above statements is/are correct?

a. 1 and 2 only

b. 3 only

c. 2 and 3 only

d. 1, 2 and 3

Solution: c

Explanation:

To portray the status of higher education in the country, Ministry of Education has

endeavoured to conduct an annual web-based All India Survey on Higher Education

(AISHE) since 2010-11. The survey covers all the Institutions in the country engaged in

imparting of higher education. It is the 10th in the series of AISHE annually released by the

Department of Higher Education.

These are useful in making informed policy decisions and research for development of

education sector.

Page 82: QUIZ COMPILATIONS – JUNE PART 1

GOALTIDE IAS ACADEMY 81

QUIZ COMPILATIONS – JUNE PART 1

Key features of All India Survey on Higher Education Report 2019-20: This only you need

to know and remember.

1. Total Enrolment in Higher Education stands at 3.85 crore in 2019-20 as compared to

3.74 crore in 2018-19, registering a growth of 11.36 lakh (3.04 %). Total enrolment

was 3.42 crore in 2014-15.

2. Gross Enrolment Ratio (GER), the percentage of students belonging to the eligible

age group enrolled in Higher Education, in 2019-20 is 27.1% against 26.3% in 2018-

19 and 24.3% in 2014-2015.

3. Gender Parity Index (GPI) in Higher Education in 2019-20 is 1.01 against 1.00 in

2018-19 indicating an improvement in the relative access to higher education for

females of eligible age group compared to males.

4. Pupil Teacher Ratio in Higher Education in 2019-20 is 26.

5. In 2019-20: Universities: 1,043(2%); Colleges: 42,343(77%) and stand-alone

institutions: 11,779(21%).

Page 83: QUIZ COMPILATIONS – JUNE PART 1

GOALTIDE IAS ACADEMY 82

QUIZ COMPILATIONS – JUNE PART 1

6. 3.38 crore Students enrolled in programmes at under-graduate and post-graduate

level. Out of these, nearly 85% of the students (2.85 crore) were enrolled in the six

major disciplines such as Humanities, Science, Commerce, Engineering &

Technology, Medical Science and IT & Computer.

7. The number of students pursuing PhD in 2019-20 is 2.03 lakh against 1.17 lakh in

2014-15.

8. The Total Number of Teachers stands at 15,03,156 comprising of 57.5% male and

42.5% female.

Link: Link: https://indianexpress.com/article/education/education-ministry-releases-aishe-

report-2019-20-number-of-phds-increased-60-in-last-five-years-7352559/

Q85. Consider the following Seas.

1. Aral Sea.

2. Red Sea

3. Caspian Sea.

Arrange the above seas from west to east.

a. 3-2-1

b. 2-3-1

c. 2-1-3

d. 3-1-2

Solution: b

Explanation:

Page 84: QUIZ COMPILATIONS – JUNE PART 1

GOALTIDE IAS ACADEMY 83

QUIZ COMPILATIONS – JUNE PART 1

Q86. Consider the following statements regarding Delimitation Commission.

1. It is formed as per the procedure mentioned in the Constitution.

2. Till now, the commission has been set up 4 times.

3. SC judge is the chairman of the Commission.

Select the correct code.

a. 1 only

b. 2 and 3 only

c. 3 only

d. 2 only

Solution: b

Explanation:

First statement is incorrect.

Article 82 of Indian Constitution provides for delimitation and it says: Upon the completion

of each census, the allocation of seats in the House of the people to the States and the

division of each State into territorial constituencies shall be readjusted by such authority

and in such manner as Parliament may by law determine.

Delimitation Commission:

Page 85: QUIZ COMPILATIONS – JUNE PART 1

GOALTIDE IAS ACADEMY 84

QUIZ COMPILATIONS – JUNE PART 1

As per Article 82, Parliament by law enacted a Delimitation Act after every census. Once

the Act comes into force, the Central Government constitutes a Delimitation Commission.

Second statement is correct.

Therefore, Delimitation Commission have been constituted four times since independence:

a. In 1952 under Delimitation Commission Act, 1952

b. In 1963 under Delimitation Commission Act, 1962

c. In 1973 under Delimitation Commission Act, 1972

d. In 2002 under Delimitation Commission Act, 2002

Third statement is correct.

What is the composition of the Delimitation Commission?

Q87. Consider the following statements.

1. The procedure for the impeachment of President is mentioned in the Constitution.

2. The proposal to impeach the President shall be preferred in House only after it is

signed by not less than one-fourth of total member of the House.

Which of the above statements is/are correct?

a. 1 only

b. 2 only

c. Both 1 and 2

d. Neither 1 nor 2

Solution: c

Explanation:

Do remember, for SC judge, it is not mentioned in the Constitution.

We will refer here article number 61. Read the articles below before proceeding to next

question. Both the statements are correct.

Page 86: QUIZ COMPILATIONS – JUNE PART 1

GOALTIDE IAS ACADEMY 85

QUIZ COMPILATIONS – JUNE PART 1

Q88. Consider the following Harrapan sites.

1. Mehrgarh

2. Kot Diji

3. Chanhudaro

4. Dholavira

5. Surkotada

6. Alamgirpur

Which of the above sites are located in today’s India?

a. 4, 5 and 6 only

b. 1, 2 and 4 only

c. 1, 3, 4 and 6 only

d. 3, 4, 5 and 6 only

Solution: a

Explanation:

Very important topic for UPSC. At least remember sites. Today, you will learn that.

Below is the list of all sites. Please look at all sites at once carefully. Just have a look. You

will remember in exam.

From the below given image, we can say only Dholavira, Alamgirpur and Surkotada are

located in India.

Page 87: QUIZ COMPILATIONS – JUNE PART 1

GOALTIDE IAS ACADEMY 86

QUIZ COMPILATIONS – JUNE PART 1

Q89. The provision coverage ratio (PCR) of all Banks increased sharply from 50 per cent in

September 2019 to 60 per cent in 2020. What does it mean?

1. It will increase the availability of money to the banks to give loans.

2. It means there was good recovery of bad loans in banks.

Which of the above statements is/are correct?

a. 1 only

b. 2 only

c. Both 1 and 2

d. Neither 1 nor 2

Solution: d

Explanation:

Read the image given below.

Page 88: QUIZ COMPILATIONS – JUNE PART 1

GOALTIDE IAS ACADEMY 87

QUIZ COMPILATIONS – JUNE PART 1

Q90. In the Gandhara school of art, Buddha is generally depicted standing frontally and one

leg is shown bent. He wore a heavy robe which covers his both shoulders and his right hand

is upraised with left hand hanging down. This posture of Buddha is called:

a. Anjali Mudra

b. Abhay Mudra

c. Bhumisparsha Mudra

d. Dharma chakra Mudra

Solution: b

Explanation:

Page 89: QUIZ COMPILATIONS – JUNE PART 1

GOALTIDE IAS ACADEMY 88

QUIZ COMPILATIONS – JUNE PART 1

Buddhas and bodhisattvas often are depicted in Buddhist art with stylized hand gestures

called mudras. The word "mudra" is Sanskrit for "seal" or "sign," and each mudra has a

specific meaning. Buddhists sometimes use these symbolic gestures during rituals and

meditation.

Abhay Mudra: The Abhay mudra is the open right hand, palm out, fingers pointing up,

raised to about the height of the shoulder. Abhaya represents the accomplishment of

enlightenment, and it signifies the Buddha immediately after his realization of

enlightenment.

Q91. Consider the following statements.

1. India is the second largest producer of silk in the world after China.

2. Among all silk verities produced in India, Mulberry accounts for more than 70

percent of total silk production.

3. More than two third of silk production is from southern states of India.

Which of the above statements is/are correct?

a. 1 only

b. 2 only

c. 2 and 3 only

d. 1, 2 and 3

Solution: d

Explanation:

News is:

Page 90: QUIZ COMPILATIONS – JUNE PART 1

GOALTIDE IAS ACADEMY 89

QUIZ COMPILATIONS – JUNE PART 1

The explanation has been taken from Annual Report of Ministry of Textiles, 2019-20. See

link in the end.

Silk is an insect fibre, with lustre, drape and strength. Because of these unique features, silk

is known as the “Queen of Textiles”, the world over. India has been the land of ancient

civilization and has contributed many things to the world, silk being one of them. India is the

second largest producer of silk in the world and also the largest consumer. Nevertheless,

India is the only country, which is producing all the five commercial varieties of silk,

namely Mulberry, Tropical Tasar, Oak Tasar, Muga and Eri.

Third statement is correct:

Karnataka is the largest producer of silk in India, accounting for more than a third of the

country’s production. Other silk producing states are Andhra Pradesh, Tamil Nadu, West

Bengal and Bihar.

Also see Central Silk Board,

Page 91: QUIZ COMPILATIONS – JUNE PART 1

GOALTIDE IAS ACADEMY 90

QUIZ COMPILATIONS – JUNE PART 1

Link: http://texmin.nic.in/sites/default/files/AR_MoT_2019-20_English.pdf

Q92. Consider the following statements regarding the Particularly Vulnerable Tribal Groups

(PVTGs):

1. The Dheobar Commission created Primitive Tribal Groups (PTGs) as a separate

category in 1970s, which was later renamed as PVTGs.

2. PVTGs resides in more than 50 percent of total number of Indian states.

3. 50 Percent of PVTGs are found in North Eastern part of India.

Select the correct code.

a. 1 and 2 only

b. 2 and 3 only

c. 1 and 3 only

d. 1, 2 and 3

Solution: a

Explanation:

In 1973, the Dhebar Commission created Primitive Tribal Groups (PTGs) as a separate

category, who are less developed among the tribal groups. In 2006, the Government of India

renamed the PTGs as Particularly Vulnerable Tribal Groups (PVTGs).

How they are identified?

Government of India designed a procedure to identify PVTGs. According to the procedure,

the state governments or UT governments submit proposals to the Central Ministry of Tribal

Page 92: QUIZ COMPILATIONS – JUNE PART 1

GOALTIDE IAS ACADEMY 91

QUIZ COMPILATIONS – JUNE PART 1

Welfare for identification of PVTGs. After ensuring the criteria is fulfilled, the Central

Ministry selects those groups as PVTGs.

Currently, 75 tribal groups have been categorized as Particularly Vulnerable Tribal Groups

(PVTG)s. PVTGs reside in 18 States and UT of A&N Islands. Second statement is correct.

The Ministry of Tribal Affairs implements the Scheme of “Development of Particularly

Vulnerable Tribal Groups (PVTGs)” exclusively for them. Now, you will go through the entire

table given below once. North East covers only a small portion of PVTGs. Go through this

table.

Q93. Which of the following is geographically closest to Great Nicobar?

a. Sumatra

b. Borneo

c. Java

d. Lakshadweep

Solution: a

Explanation:

Very easy, if you have printed the map in your mind.

Q94. In the context of India, ‘Right to know’ about the assets and qualifications of

contesting candidates is a:

a. Fundamental Right

b. Constitutional Right

c. Legal Right

d. Doesn’t confer any right

Solution: a

Page 93: QUIZ COMPILATIONS – JUNE PART 1

GOALTIDE IAS ACADEMY 92

QUIZ COMPILATIONS – JUNE PART 1

Explanation:

The court in its judgment in Union of India v. Association for Democratic Reforms & Another

(2002) held that every voter had a fundamental right to know, under Article 19(1)(a), the

assets and qualifications of candidates because it was only on receipt of such information

that they could express themselves by exercising their vote for the best candidate.

Q95. Consider the following statements.

1. There should not be a gap of more than six months between two sessions of

Parliament.

2. A notice of the motion to remove Judge of Supreme Court shall be accepted by

Speaker after it is signed by 100 members of Lok Sabha.

Which of the following provisions are mentioned in the Constitution?

a. 1 only

b. 2 only

c. Both 1 and 2

d. Neither 1 nor 2

Solution: a

Explanation:

By convention, Parliament meets for three sessions in a year. The Constitution does not

specify when or for how many days Parliament should meet.

Article 85 only requires that there should not be a gap of more than six months between

two sessions of Parliament. This is part of Constitution.

Parliament enacted The Judges (Inquiry) Act, 1968. There it is mentioned that it should be

signed by 100 members of Lok Sabha. Read below. It is not mentioned in the Constitution.

Page 94: QUIZ COMPILATIONS – JUNE PART 1

GOALTIDE IAS ACADEMY 93

QUIZ COMPILATIONS – JUNE PART 1

Q96. Which of the following temples is dedicated to Lord Shiva?

1. Venkateshwara Temple, Tirupati

2. Mahakaleshwar Temple, Ujjain

3. Jagannath Temple, Puri

4. Ekambareswarar Temple, Tamil Nadu

Select the correct code.

a. 1, 2 and 3 only

b. 1 and 4 only

c. 2 and 4 only

d. 2, 3 and 4 only

Solution: c

Explanation:

Venkateswara Temple is an important Vaishnavite temple situated in the hill town of

Tirumala in Chittoor district of Andhra Pradesh, India. The Temple is dedicated to Lord

Venkateswara, an incarnation of Vishnu.

Ekambareswarar Temple is a Hindu temple dedicated to the deity Shiva, located in the

town of Kanchipuram in Tamil Nadu, India.

About Mahakaleshwar Temple: Situated on the banks of the holy river Shipra in Ujjain, the

Mahakaleshwar Temple dedicated to Lord Shiva is one of the most famous shrines in the

country.

Page 95: QUIZ COMPILATIONS – JUNE PART 1

GOALTIDE IAS ACADEMY 94

QUIZ COMPILATIONS – JUNE PART 1

Q97. In the context of the location of industries, the supply of raw material is the most

important factor for which of the following industries?

a. Automobile Industry

b. Electronic Goods Industry

c. Sugar Industry

d. Cotton Textile Industry

Solution: c

Explanation:

Industries using weight-losing raw materials are located in the regions, where raw materials

are located. Thus, sugar mills in India are located in sugarcane growing areas. Sugarcane

needs to be supplied to mills within 24 hours of cutting to ensure better recovery of sugar.

Hence, the location of Sugar mills is near the sugarcane producing areas. Similarly, the pulp

industry, copper smelting and pig iron industries are located near their raw materials.

Options (a) and (b) are examples of weight gaining industry, and they require more diverse

ancillary material. Hence, these are generally located near the market. Cotton textile is not

a weight losing industry. The raw material is not the most dominant factor, but a humid

climate, cheap labour and transport and most importantly, a market for trade determine the

location of the cotton textile industry. Therefore, the correct answer is (c).

Q98. In the context of temple architectures, Keralantakam, Rasarasan and Tiruanukkam

represents:

a. Separate kitchen areas for different group of people inside Virupaksha Temple

b. different religious places in the temple to perform different religious activities during

Guptas

c. Places dedicated for waste management of Temple during the reign of Vijayanagar

d. Three Main entrances to the shrine in Brihadisvara Temple

Solution: d

Explanation:

Brihadisvara Temple: some important features of temple.

Page 96: QUIZ COMPILATIONS – JUNE PART 1

GOALTIDE IAS ACADEMY 95

QUIZ COMPILATIONS – JUNE PART 1

Q99. In the context of Indian Polity, consider the following statements regarding Tenth

Schedule.

1. A person shall be disqualified for being a member of the Legislative Assembly or

Legislative council of a state, if he is so disqualified under the Tenth schedule.

2. The purpose which the Tenth Schedule seeks to achieve were explained in Kihoto

Hollohan v. Zachillhu and Others case.

3. Tenth schedule was passed during priministership of Rajiv Gandhi.

Select the correct code.

a. 1 and 2 only

b. 2 and 3 only

c. 1 and 3 only

d. 1, 2 and 3

Page 97: QUIZ COMPILATIONS – JUNE PART 1

GOALTIDE IAS ACADEMY 96

QUIZ COMPILATIONS – JUNE PART 1

Solution: d

Explanation:

First statement is correct.

Clause (2) of Article 191 states, "A person shall be disqualified for being a member of the

Legislative Assembly or Legislative council of a state, if he is so disqualified under the Tenth

schedule".

Second statement is also correct. Just remember, Kihoto Hollohan v. Zachillhu and Others,

1992 case is related to tenth schedule.

The underlying object and the purpose which the Tenth Schedule seeks to achieve were

explained as under in Kihoto Hollohan v. Zachillhu and Others, 1992:

“These provisions in the Tenth Schedule give recognition to the role of political parties in the

political process. A political party goes before the electorate with a particular programme

and it sets up candidates at the election on the basis of such programme. A person who gets

elected as a candidate set up by a political party is so elected on the basis of the programme

of that political party. The provisions of Paragraph 2(1)(a) proceed on the premise that

political propriety and morality demand that if such a person, after the election, changes his

affiliation and leaves the political party which had set him up as a candidate at the election,

then he should give up his membership of the legislature and go back before the electorate.

The same yardstick is applied to a person who is elected as an Independent candidate and

wishes to join a political party after the election”.

Rajiv Gandhi was the Prime Minister when 10th schedule was passed.

Page 98: QUIZ COMPILATIONS – JUNE PART 1

GOALTIDE IAS ACADEMY 97

QUIZ COMPILATIONS – JUNE PART 1

Q100. Consider the following Nations.

1. Norway

2. Sweden

3. North Korea

Which of the above nations share boundaries with Russia?

a. 1 and 2 only

b. 2 and 3 only

c. 1 and 3 only

d. 1, 2 and 3

Solution: c

Explanation:

The countries with whom Russia shares its land border include North Korea, China, Norway,

Finland, Ukraine, Kazakhstan, Poland, Georgia, Mongolia, Latvia, Estonia, Azerbaijan,

Belarus, and Lithuania.

Page 99: QUIZ COMPILATIONS – JUNE PART 1

GOALTIDE IAS ACADEMY 98

QUIZ COMPILATIONS – JUNE PART 1

Q101. Consider the following statements.

1. A member of a house belonging to any political party shall be disqualified if he or she

voluntarily given up membership of such political party.

2. In the situation where one-thirds of the legislators of a political party decide to merge

into another party, neither the members who decide to join nor the ones who stay

with the original party will face disqualification.

3. Any question regarding disqualification arising out of defection is to be decided by the

presiding officer of the House.

4. Tenth Schedule was enacted during priministership of Indira Gandhi.

Which of the above statements is/are incorrect?

a. 1 and 2 only

b. 2 and 4 only

c. 1, 2 and 4 only

d. 4 only

Solution: b

Explanation:

First statement is correct:

The Tenth Schedule of the constitution, which is popularly referred to as the anti-defection

law, says, under the heading of “disqualification on ground of defection”, that a member of

a house belonging to any political party shall be disqualified if he or she has a) voluntarily

given up membership of such political party; or (b) votes or abstains from voting in such

House contrary to any direction issued by the political party to which he belongs.

Second statement is incorrect: Its two-third.

Third statement is correct:

Decision on questions as to disqualification on ground of defection. - If any question arises

as to whether a member of a House has become subject to disqualification under this

Schedule, the question shall be referred for the decision of the Chairman or, as the case

may be, the Speaker of such House.

Page 100: QUIZ COMPILATIONS – JUNE PART 1

GOALTIDE IAS ACADEMY 99

QUIZ COMPILATIONS – JUNE PART 1

Originally, the Act provided that the presiding officer’s decision was final and could not be

questioned in any court of law. But, in Kihoto Hollohan case (1993), the Supreme Court

declared this provision as unconstitutional on the ground that it seeks to take away the

jurisdiction of the SC and the high courts.

Fourth statement is incorrect:

Prime Minister was RAJIV GANDHI.

Q102. Consider the following statements.

1. MSP for paddy is less than MSP for Ragi, Jowar, Bajra (not combined).

2. Cost of Production is the only important factor which determines MSP of crop.

3. The concept of MSP started in India after LPG reforms.

4. Government is legally bound to pay MSP even if open market rates for the said

produce are ruling below their announced floor prices.

Which of the above statements is/are correct?

a. 1 only

b. 2 and 3 only

c. 1, 3 and 4 only

d. 1 and 4 only

Solution: a

Explanation:

First statement is correct:

Page 101: QUIZ COMPILATIONS – JUNE PART 1

GOALTIDE IAS ACADEMY 100

QUIZ COMPILATIONS – JUNE PART 1

Second statement is incorrect:

Determinants Of MSP

While recommending price policy of various commodities under its mandate, the Commission keeps

in mind the various Terms of Reference given to CACP in 2009. Accordingly, it analyses

Page 102: QUIZ COMPILATIONS – JUNE PART 1

GOALTIDE IAS ACADEMY 101

QUIZ COMPILATIONS – JUNE PART 1

Third statement is incorrect:

The MSP based system has its origin in the rationing system that was introduced by the

British during the World War-II. In the year 1942, a food department was introduced by the

British administration which was later upgraded into the ministry of food.

In the mid-60s, India faced acute food shortages and started shoring up its food reserves.

The concept of MSP was first mooted in 1966-67, during green revolution, to incentivise

the Punjab farmers.

In the year 1964, Food Corporation of India (FCI) was established to procure food grains at

remunerative prices from the farmers. In 1965, the Agriculture Prices Commission (APC) was

established to regulate the pricing of procured food grains. In 1985, the APC was renamed

as Commission for Agricultural Costs & Prices (CACP).

Fourth statement is incorrect:

The Government has fixed MSPs for 23 crops-- 7 cereals (paddy, wheat, maize, bajra, jowar,

ragi and barley), 5 pulses (chana, arhar/tur, urad, moong and masur), 7 oilseeds (rapeseed-

mustard, groundnut, soybean, sunflower, sesamum, safflower and nigerseed) and 4

commercial crops (cotton, sugarcane, copra and raw jute).

However, the Government is not legally bound to pay MSP even if open market rates for

the said produce are ruling below their announced floor prices.

Q103. Consider the following statements regarding United Nation General Assembly.

1. Each Member State has one vote.

2. The five permanent members of the Security Council cannot hold the office of the

President of General Assembly.

3. The President of General Assembly is the final authority for any decision making in

General Assembly.

Which of the above statements is/are correct?

a. 1 and 2 only

b. 2 and 3 only

c. 1 only

Page 103: QUIZ COMPILATIONS – JUNE PART 1

GOALTIDE IAS ACADEMY 102

QUIZ COMPILATIONS – JUNE PART 1

d. 1, 2 and 3

Solution: a

Explanation:

Established in 1945 under the Charter of the United Nations, the General Assembly

occupies a central position as the chief deliberative, policymaking and representative organ

of the United Nations. Comprised of all 193 Members of the United Nations, it provides a

unique forum for multilateral discussion of the full spectrum of international issues covered

by the Charter. It also plays a significant role in the process of standard-setting and the

codification of international law.

All UN Member States are represented in the General Assembly. Each Member State has

one vote. First statement is correct.

How President is elected?

Any Member State can put forward a candidate for President of the General Assembly

(PGA). He/she is not required to be, but always has been, a citizen of the Member State

presenting the candidacy. The PGA is elected in his/her personal capacity and for the

duration of the term of office represents the membership as a whole.

The Member State of the PGA cannot at the same time hold the office of Vice-President or

Chair of a Main Committee. Thus, the five permanent members of the Security Council,

who are always Vice-Presidents, cannot hold the office of the PGA. Second statement is

correct.

What is the role and mandate of the PGA?

The PGA is the guardian of the GA Rules of Procedure but has no say in the actual

decision-making of the GA – in fact, the PGA does not have a vote in the GA. Even on

procedural matters, the PGA always remains under the authority of the GA. Third statement

is incorrect.

Q104. Rakesh Sarwal Committee is related to:

a. Issues faced by MSME

b. Electric Vehicles.

c. Education reforms post COVID.

d. Ethanol Blending in India

Solution: d

Explanation:

The central government has released an expert committee report on the Roadmap for

Ethanol Blending in India by 2025. The roadmap proposes a gradual rollout of ethanol-

blended fuel to achieve E10 fuel supply by April 2022 and phased rollout of E20 from April

2023 to April 2025.

Page 104: QUIZ COMPILATIONS – JUNE PART 1

GOALTIDE IAS ACADEMY 103

QUIZ COMPILATIONS – JUNE PART 1

Q105. Which of the following are the transnational rivers between India and Bangladesh?

1. Teesta

2. Feni

3. Gumti

4. Mahananda

Select the correct answer using the code given below:

a. 1, 2 and 3 only

b. 1 and 2 only

c. 1, 2 and 4 only

d. 1, 2, 3 and 4

Solution: d

Explanation:

Teesta River is a 315 km (196 mi) long river that rises in the eastern Himalayas, flows

through the Indian States of West Bengal and Sikkim through Bangladesh and enters the

Bay of Bengal.

Feni River is a river in the Indian State of Tripura and southeastern Bangladesh. It is a

trans-boundary river with an ongoing dispute about water rights. The Feni River originates

in South Tripura district and flows through Sabroom town and then enters Bangladesh.

Gumti is a river flowing through the north-eastern Indian state of Tripura and Bangladesh.

Page 105: QUIZ COMPILATIONS – JUNE PART 1

GOALTIDE IAS ACADEMY 104

QUIZ COMPILATIONS – JUNE PART 1

The Mahananda River is a trans-boundary river that flows through the Indian States of

West Bengal and Bihar, and Bangladesh.

Q106. Consider the following Pairs.

National Parks River Flowing through it.

1. Silent Valley National Park Kunthipuzha River

2. Valmiki wildlife sanctuary Gandak River

3. Mukundra Hills National Park Narmada River

Which of the above pairs is correctly matched?

a. 1 only

b. 1, 2 and 3 only

c. 1 and 2 only

d. 2 only

Solution: c

Explanation:

It is a difficult question. In exam, attempt only if your other questions don’t go well.

National Parks River Flowing through it.

1. Silent Valley National Park Kunthipuzha River

2. Valmiki wildlife sanctuary Gandak River

3. Mukundra Hills National Park Chambal River

Q107. Which of the following statements was not the primary reason to justify the choice of

Salt Satyagraha for beginning the Civil Disobedience Movement?

a. It offered urban populace the opportunity of a symbolic identification with mass

suffering.

b. It had no socially divisive implications for Indian society.

c. Removal of Salt Tax was aimed at saving millions of Indians from starvation.

d. It was a taxed commodity that every Indian across all castes and classes could relate

to.

Solution: c

Explanation:

Salt afforded a tiny but psychologically important income, like khadi, for the poor through

self-help. Like khadi, it offered to the urban populace the opportunity of a symbolic

identification with mass suffering. So, statement (a) is correct.

Salt in a flash linked the ideal of swaraj with a most concrete and universal grievance of the

rural poor (and with no socially divisive implications like a no-rent campaign). So, statement

(b) is correct.

Page 106: QUIZ COMPILATIONS – JUNE PART 1

GOALTIDE IAS ACADEMY 105

QUIZ COMPILATIONS – JUNE PART 1

Gandhiji wanted to make the people aware of the oppressive policies which in this case

were the ban on manufacturing of salt. And saving people from starvation was not its

primary objective. So, statement (c) is not correct.

Salt was a commodity that every Indian could relate to, be it poor, middle class, rich, Dalits

or brahman all used salt alike. Thus, it acted as a unifying cause. So, statement (d) is correct.

Therefore, the answer is (c).

Q108. Which of the following is not a part of the Tripitakas?

a. Therigatha

b. Dhammapada

c. Suttavibhaga

d. Acharanga Sutra

Solution: d

Explanation:

Tripitaka is the Buddhist canon written in Pali. It contains three parts - Vinaya Pitaka, Sutta

Pitaka and Abhidhamma Pitaka. It contains various books.

Therigatha, a part of Sutta Pitaka, consists of 73 poems — in which the early nuns

(bhikkhunis) recount their struggles and accomplishments along the road to arahantship.

Dhammapada is a collection of sayings of the Buddha in verse form. It is also a part of Sutta

Pitaka.

Suttavibhaga is the book in Vinaya Pitaka and contains rules for the behaviour of monks.

They are called as Patimokkha rules. It is divided into two parts, covering the rules for

monks and nuns, respectively.

Acharanga Sutra is the first of the twelve Angas, part of the agamas (religious texts) which

were compiled based on the teachings of Mahavira. Thus, it belongs to Jainism.

Q109. In the context of India Polity, consider the following statements regarding

Referendum.

1. As per the Constitution, Referendum is prohibited in India.

2. India has never exercised Referendum after independence.

3. Referendum is an example of indirect democracy majorly practiced by non-

democratic nations.

Which of the above statements is/are incorrect?

a. 1 and 2 only

b. 2 and 3 only

c. 2 only

d. 1, 2 and 3

Solution: d

Page 107: QUIZ COMPILATIONS – JUNE PART 1

GOALTIDE IAS ACADEMY 106

QUIZ COMPILATIONS – JUNE PART 1

Explanation:

It is asked incorrect. Be careful.

The statement 3 is incorrect as Referendum is a tool of direct democracy.

What is the difference between direct and indirect democracy?

Majority of democratic nations across the world, follow the system of an indirect

democracy, wherein the citizens elect their representatives, who in turn make laws,

determine policies and carry-on administrative work. On the other hand, in some countries

citizens directly take part in the administration of the country, decide on policy issues and

make laws as well. This form is called a direct democracy and a referendum is a

facet/means of it. In a referendum, the citizens are called to vote in person (rather than

through their elected representatives) on a policy decision to be taken by the government

(for instance, United Kingdom's decision to leave the European Union) or an amendment to

their Constitution. Several nations have incorporated provisions for referendums in their

Constitution.

Second statement is incorrect.

India has Practiced referendum after Independence. Check below:

Independent India and Referendums

The Constitution as adopted by the Constituent Assembly, did not have any provisions for a

referendum. However, the nation witnessed referendums on five key occasions.

a. First, during the incorporation of Chandernagore within the territory of India

b. Incorporation of princely state of Junagadh, wherein the citizens voted to accede to

India rather than Pakistan in the year 1948

c. Pondicherry (a former French territory) voting to join the Indian Union in the year

1954.

d. Incorporation of Goa, Daman and Diu. It should be noted that the government did

not call the this exercises a 'referendum' but an Opinion Poll. However, in effect it

was a referendum.

e. The last referendum happened in the year 1975, wherein the inhabitants of Sikkim

decided on their merger with India

First statement is also incorrect.

The Constitution of India is silent on a referendum, therefore theoretically a referendum is

neither allowed nor prohibited.

Q110. Which of the following aspects can lead to appreciation in the exchange rate of the

Rupee?

1. Increase in foreign tourists coming to India.

2. Indian citizens investing abroad.

3. A decrease in crude oil prices.

Page 108: QUIZ COMPILATIONS – JUNE PART 1

GOALTIDE IAS ACADEMY 107

QUIZ COMPILATIONS – JUNE PART 1

4. Increase in exports by Indian pharmaceutical companies.

Select the correct answer using the code given below:

a. 1 and 2 only

b. 1, 3 and 4 only

c. 2 and 4 only

d. 1 and 3 only

Solution: b

Explanation:

Increase in foreign tourists coming to India would increase the demand for Rupee. This

would lead to an appreciation in the rupee value. So, statement (1) is correct.

Indian citizens investing abroad would demand foreign currency in exchange for Rupee.

This would lead to the price of the foreign currency increasing relative to the rupee,

leading to a depreciation in the value of the rupee. So, statement (2) is not correct.

A decrease in crude oil prices would reduce the import bill of India as India imports about

70 per cent of its crude requirements. This would reduce the demand for foreign currency,

which in turn would lead to an increase in the value of the rupee. So, statement (3) is

correct.

Increase in exports by Indian pharmaceutical companies would bring foreign exchange in

India. The price of foreign currency would reduce in terms of the rupee. This would lead to a

depreciation in the value of the foreign currency and an appreciation in the value of the

rupee. So, statement (4) is correct.

Therefore, the correct answer is (b).

Page 109: QUIZ COMPILATIONS – JUNE PART 1

GOALTIDE IAS ACADEMY 108

QUIZ COMPILATIONS – JUNE PART 1